Clinical Cases in Neurology [1st ed.] 978-3-030-16627-4;978-3-030-16628-1

This book presents a selection of neurological clinical cases and tests the reader in order to allow them to gain clinic

1,593 171 8MB

English Pages XXV, 188 [198] Year 2019

Report DMCA / Copyright

DOWNLOAD FILE

Polecaj historie

Clinical Cases in Neurology [1st ed.]
 978-3-030-16627-4;978-3-030-16628-1

Table of contents :
Front Matter ....Pages i-xxv
Anticoagulation and Fall (Ondrej Dolezal)....Pages 1-6
Pain in the Face (Ondrej Dolezal)....Pages 7-12
Dementia and “Something Else” (Ondrej Dolezal)....Pages 13-20
New Symptoms in Old Illness (Ondrej Dolezal)....Pages 21-26
Young Lady with Severe Arm Pain (Ondrej Dolezal)....Pages 27-31
“Remember Bingo Numbers!” (Ondrej Dolezal)....Pages 33-38
Poor Hearing and Seizures (Ondrej Dolezal)....Pages 39-46
Tired and Sleepy Woman with Headaches (Ondrej Dolezal)....Pages 47-51
Disorientation, Confusion I (Ondrej Dolezal)....Pages 53-58
Disorientation, Confusion II (Ondrej Dolezal)....Pages 59-64
Woman with Sudden Headache (Ondrej Dolezal)....Pages 65-71
First Seizure (Ondrej Dolezal)....Pages 73-79
Elephant in the Room vs. Red Herring (Ondrej Dolezal)....Pages 81-89
Gait Disorder and Falls (Ondrej Dolezal)....Pages 91-97
“Tongue Tied” Man with Opiate Addiction (Ondrej Dolezal)....Pages 99-105
Right Sided Hemiparesis in Young Smoker (Ondrej Dolezal)....Pages 107-112
Young Man with Pins and Needles (Ondrej Dolezal)....Pages 113-118
Painful and Weak Young Man (Ondrej Dolezal)....Pages 119-123
Recurrent Otitis Media (Ondrej Dolezal)....Pages 125-131
Deafness and Shoulder Weakness (Ondrej Dolezal)....Pages 133-139
Young Girl with the Wrist Drop (Ondrej Dolezal)....Pages 141-145
Facial Tingling and Poor Hearing (Ondrej Dolezal)....Pages 147-153
Balance and Gait Problems (Ondrej Dolezal)....Pages 155-160
Numb Legs and Constipation (Ondrej Dolezal)....Pages 161-165
Yellow and Weak Young Man (Ondrej Dolezal)....Pages 167-170
Back Matter ....Pages 171-188

Citation preview

In Clinical Practice

Ondrej Dolezal

Clinical Cases in Neurology

In Clinical Practice

Taking a practical approach to clinical medicine, this series of smaller reference books is designed for the trainee physician, primary care physician, nurse practitioner and other general medical professionals to understand each topic covered. The coverage is comprehensive but concise and is designed to act as a primary reference tool for subjects across the field of medicine. More information about this series at http://www. springer.com/series/13483

Ondrej Dolezal

Clinical Cases in Neurology

Ondrej Dolezal Department of Neurology Dumfries and Galloway Royal Infirmary Dumfries, UK

ISSN 2199-6652     ISSN 2199-6660 (electronic) In Clinical Practice ISBN 978-3-030-16627-4    ISBN 978-3-030-16628-1 (eBook) https://doi.org/10.1007/978-3-030-16628-1 © Springer Nature Switzerland AG 2019 This work is subject to copyright. All rights are reserved by the Publisher, whether the whole or part of the material is concerned, specifically the rights of translation, reprinting, reuse of illustrations, recitation, broadcasting, reproduction on microfilms or in any other physical way, and transmission or information storage and retrieval, electronic adaptation, computer software, or by similar or dissimilar methodology now known or hereafter developed. The use of general descriptive names, registered names, trademarks, service marks, etc. in this publication does not imply, even in the absence of a specific statement, that such names are exempt from the relevant protective laws and regulations and therefore free for general use. The publisher, the authors, and the editors are safe to assume that the advice and information in this book are believed to be true and accurate at the date of publication. Neither the publisher nor the authors or the editors give a warranty, express or implied, with respect to the material contained herein or for any errors or omissions that may have been made. The publisher remains neutral with regard to jurisdictional claims in published maps and institutional affiliations. This Springer imprint is published by the registered company Springer Nature Switzerland AG The registered company address is: Gewerbestrasse 11, 6330 Cham, Switzerland

To my children, Tomas, Lukas and Krystof, and most of all to my inspiring wife, Jitka (who has to put up with us all).

Foreword and Acknowledgements Most of all, I want to thank my patients, as through them I had the opportunity to meet some of the most extraordinary people. It would not be possible to write this book without all my colleagues I met over the last two decades and who passed some of their experiences and advices onto me. Scenarios/ reports in this book are compilation of cases I collected over many years. I would like to thank my teachers and colleagues at the Department of Neurology and Center of Clinical Neuroscience at First Faculty of Medicine of Charles University and General University Hospital in Prague. Where I studied and worked and where I was honoured to be briefly one of the teachers as well. I have to extend my thanks to my colleagues and friends at Buffalo Neuroimaging Analysis Center, where I spent an unforgettable year of my life. Without them, the MRI and ultrasound would mean just “black and white grainy pictures” to me. Last but not least, I would like to express my gratitude to my colleagues at Dumfries and Galloway Royal Infirmary as without their day-to-day support I would not be able to survive as their clinical expertise helped me in diagnostic despair. My appreciation also belongs to my colleagues at the Department of Clinical Neurosciences (DCN) at the Western General Hospital and most recently at the Royal Infirmary in Edinburgh. Special thanks I reserve for general practitioners in Dumfries and Galloway whose experience, skills and broad knowledge are always an inspiration for a neurologist like me. vii

viii

Foreword and Acknowledgements

The following have provided extremely valuable ideas and advices: L.  Clark and F.  Green (endocrinology and general medicine); G.  Jones (infectious diseases and general medicine); M.  McMahon (rheumatology and general medicine); F.  Haque, D.  Hill, R.  Mcdonald, P.  Hrobar, P.  Handrk, P. Keston, and J. Downer (radiology); S. Saha and A. Wahab (gastroenterology and general medicine); T.  Muniraju and K. Donaldson (renal and general medicine); R. Thomas (haematology); S. Finlay and M. Polak (acute medicine); P. Nytrova and E. Nespor (neurology); G. Moran and C. Lakomy (neurology and neurophysiology); I. Hunter (ophthalmology) and E. Bastow (editing). Dumfries, UK

Ondrej Dolezal, PhD, FRCP

Introduction

It is fair to start with one question. Why would it be useful to write a book full of case reports and clinical scenarios? As many of you, I have always loved case reports. They cover clinical practice point of view and also improve your knowledge about the subject/diagnosis in question. Case reports are usually describing new or unusual presentation, rare complication or surprising outcome. Special kind of case reporting is a “rare disease” case. These cases are relatively irritating, because you can guess you would not be lucky enough to see similar case in your day-to-day practice. But it is true that you will always remember the case of a “man with broken heart”, describing myocardial rhabdomyosarcoma or the case of Partington syndrome (upper body dystonia, developmental delay, etc.) caused by mutation in ARX gene. However, it is not relevant if the illness is common or rare, and diagnosis is always based on initial clinical picture, carefully taken history and objective examination and eventually concluded by well-targeted tests. Diagnostic thoughts and diagnostic processes are always based on clinical story (history), clinical examination and test results. So it stands on triad of “symptom-sign-syndrome” if you like. In teaching and educational role, you very often encounter junior colleagues with excellent academic and theoretical knowledge but who struggle to connect all loose ends in this logical “chain of thought”. I believe that specialised doctors, including me, are all making one mistake. We are approaching our peers from different specialties, students and junior doctors as them being also ix

x

Introduction

specialists, in my case as neurologists or at least ­neurologists in training. When I was a medical student, I remember memorising brainstem syndromes (Wallenberg, Dejerine, Benedikt, Claude, Weber, Nothnagel, Parinaud, Millard-­Gubler, Avellis, Tapia, Vernet, etc.) for my neurology exam. It is fair to say that I have used some of that knowledge in the future as a neurologist, but from approximately 100 graduates at the end of my studies, only 5 or 6 eventually chose neurology or neurosurgery as primary specialty. It means that for majority of my peers, memorising this very particular and detailed information proved just to be a pointless exercise. In my clinical practice, I am also responsible for referral triage. Sometimes, I am surprised by the lack of logic in differential diagnostic thinking. Frequently, my colleagues struggle differentiating upper and lower motor neuron level of lesion (central vs. peripheral pathology). On asking medical students, junior doctors and colleagues of other specialties why they are not more comfortable with neurological examination, they all mentioned its complexity (and I quote: “with all your signs, Latinisms and French or German names”) and lack of time and capacity to change it (quote: “it would be nice to have enough time to refresh my neurological skills”). After listening to this, I had to admit one thing: while I am as a neurologist entitled to have endless discussion with other neurologists about the presence of Strumpel, Hoffmann, Roche, Tromner, Chadock, etc., to discuss if symptoms originate from the vermis or hemispheres of the cerebellum or to discuss rigidity vs. spasticity conundrum, for the rest of our colleagues and trainees, plantar response (Babinski), identifying correctly the finger/nose and gait ataxia and recognising increased muscle tone would suffice. It was later in my career that I adopted stick figure diagram to summarise neurological examination, and my thanks belong to William E. DeMyer’s Technique of the Neurological Examination. Over the years, I simplified it even more, but this diagram of neurological examination proved to be (a) easier to record and (b) easier to interpret. With an advance of technology, we tend to rely on fancy scanners (imaging in general) and biochemical, immunological and genetic tests. Thanks to these developments we can deliver diagnosis more precisely and sometimes promptly. But

Introduction

xi

all these results have to be always put into a clinical context. So in somebody, “empty sella” on MRI means absolutely nothing, but in somebody else, it can be suggestive of increased intracranial pressure. It is only the clinician who can decide. Another well-known example is “hyperintensities” (white dot/s) on MRI (especially on FLAIR sequences). Depending on age, distribution of these lesions and mainly clinical picture, only the clinician can make decision if we are dealing with demyelination, vascular small vessel changes, postinfectious, post-traumatic changes and migraines or if these findings are within normal limits. Diagnosis of many neurological diseases is entirely based on clinical picture, and other tests (MRI, electrophysiology (EEG), etc.) are having marginal and only supportive role. We can mention here migraines, epilepsy and more increasingly unexplained neurological symptoms (functional, dissociative symptoms). The picture below shows ideal diagnostic process put into the context of this book with one clinical example (stroke patient). Picture: How to approach cases; using this book (left column), in general (middle column) with clinical example (right column)? First section introducing the case & Neurological examination

Thoughts

Personal medical history Clinical story

Sudden onset of right sided weakness, speech disturbance

Neurological examination

right hemiparesis, global aphasia, (UMN) right VII palsy

Differential diagnosis

Stroke (ischaemic vs. hemorhagic), postictal paresis less likely

Diagnostic plan Indication of tests

CT brain – urgent Blood tests,

Analysing results in the clinical context

Normal, but history less then 90 minutes

Formulating diagnosis, Treatment

Ischaemic stroke likely, thrombolysis

Recording interesting aspect of the case/further research

Patient recovered well, good response to treatment

Answers

Hallmarks Further reading Notes

xii

Introduction

In this book, the reader will be provided with a very short history description and brief neurological examination with diagram. At that point, the reader can think what can be responsible for symptoms and confront her/his ideas with “Thoughts” section which follows. The most appropriate test(s) will be chosen. Afterwards, readers can look at the scan or other test results and try to interpret it themselves. Imaging is deliberately left without a legend. After that, section called “Answers” follows, and each case is concluded by definite diagnosis and short summary of outcome and treatment. In the section “Hallmarks”, you can go again through learning points, interesting twists and even mistakes made. “Further reading” section provides articles which described similar clinical situation and also pregraduate/postgraduate textbooks where readers can refresh their knowledge of ­general principles, epidemiology and therapy. At the end of the book, there is a “Key” with miniatures of key scan or scans (if the scan was relevant) for each case which reader can use for confrontation. Before you start reading this book, I would like to mention one important issue. It would be absolutely tragic if the main conclusion of the reader would be that imaging is always the answer. In fact, it is rather the opposite. Emphasis is mainly on history and clinical examination (in its simplified form), but I deliberately chose cases which were “cracked” by imaging (with one or two exceptions when scan was actually indicated questionably). From didactic point of view, conclusive scan as “jewel in the crown” seemed to provide prompt answer. I believe that it also allows cases to be remembered better. This book is not trying to cover theoretical, academic or research neurology and should be used as purely a tool helping with diagnostic thoughts. Saying all the above, target audience would be medical students, junior doctors and colleagues of other specialties than neurology. This book with all simplification mentioned above and below (e.g. stick figure diagram) would be of limited value to neurologists and neurosurgeons. With one exemption. Especially when there was lack of suitable patients on wards, I used this book with success in bed-side teaching (introducing the case, describing

Introduction

xiii

exam to students and then letting students to show their diagnostic qualities). Discussion about the case, usually very detailed, was the result. You can use the “Notes” section for remarks, comments and hallmarks. You can record similar cases you have seen in your practice or add some other comments which would make future use of particular scenario more simple (e.g. mentioning where junior doctors misjudged the case, or what students found surprising or difficult to understand, interpret, etc.). The following cases do not necessarily reflect individual patients, and they have to be interpreted rather as scenarios; saying that, no symptoms were added or removed, and cases showed here were inspired by real patients.

Stick Figure Diagram Stick figure diagram represents simplified version of neurological examination, however agreeing with level of emergency department and general practice templates. The table below summarises abbreviations.

Reflexes −

Not distinguishing biceps, triceps, brachioradialis in upper extremities Absent

+

Present

++

Increased

+++

Clonus

Muscle strength 1/5-5/5

Muscle strength using range from no contraction to normal

Plantars Negative (Neg.)/ positive (Pos.)

Down-going/flexor, up-going/extensor

CN

Cranial nerves

xiv

Introduction CN, cerebellar normal

+

+

5/5

5/5

+

+

+ Neg. 5/5

+ Neg. 5/5

Normal examination would look like this diagram on the left. So reflexes are present in the upper extremities; if one of the reflexes (biceps, triceps, brachioradialis) is outstanding, it would be mentioned in the text. Muscle strength in upper limbs is normal (5/5). Patellar reflex/ quadriceps/knee jerk and Achilles tendon reflex/ankle jerk are present and normal, plantars are flexor (negative), and leg strength is normal (5/5). There is no sensory deficit and sphincter functions are normal as well. If there was any particular muscle group weakness (innervated by peroneal, tibial, radial, etc.), it would be described in the text.

Introduction

xv

CN, cerebellar functions normal

5/5

5/5

+ +

+ +

5/5

5/5

+ +

+ +

– Neg. 4+/5

– Neg. 5/5

You can see another example on the left. Sensory deficit (marked as dash lines across extremities) in our diagram is not discriminating between negative and positive sensory symptoms. The same is valid for sphincter symptoms as the diagram just shows (using X symbol) their presence, irrespective of retention, incontinence, overflow incontinence, urgency, etc. Details are again mentioned in the text if needed. The example below shows patient with bilateral leg paraesthesias or hypaesthesia/numbness and incontinence or urgency or retention. There are also brisk/increased reflexes in arms (++), right leg is weaker (4+/5), and patellar reflex is increased on both sides (++), with bilateral absent ankle reflex (−).

xvi

Introduction

For teaching purposes, I would like to encourage teachers/lecturers to modify these cases according to their needs. Based on specific educational requirements, teachers can accent history, examination, radiology aspects or treatment.

Abbreviations

ACE Angiotensin-converting enzyme ACRA Acetylcholine receptor antibodies AIDS Acquired immune deficiency syndrome ALS Amyotrophic lateral sclerosis ANA Antinuclear antibody ANCA Anti-neutrophil cytoplasmic antibodies AVM Arteriovenous malformation CIS Clinically isolated syndrome CK Creatine kinase CLL Chronic lymphocytic leukaemia CMV Cytomegalovirus CN Cranial nerves COPD Chronic obstructive pulmonary disease CRP C-reactive protein CSF Cerebrospinal fluid CT Computed tomography DAVF Dural arteriovenous fistula DMT Disease-modifying therapies DSA Digital subtraction angiography EBV Epstein-Barr virus ECG Electrocardiogram EEG Electroencephalogram EMG Electromyography ENA Extractable nuclear antigen EOMs Extra-ocular muscles ESR Erythrocyte sedimentation rate FLAIR Fluid-attenuated inversion recovery (type of T2 MRI sequence) xvii

xviii

Abbreviations

FTD GBS GCS GP HIV HMSN HNPP

Fronto-temporal dementia Guillain-Barré syndrome Glasgow Coma Scale General practitioner Human immunodeficiency virus Hereditary motor and sensory neuropathy Hereditary neuropathy with liability to pressure palsy HSV Herpes simplex virus IAM Internal auditory meatus ICP Intracranial pressure INR International normalised ratio IVIG Intravenous immunoglobulin JME Juvenile myoclonic epilepsy LFT Liver function tests LMWH Low-molecular-weight heparin LP Lumbar puncture MDT Multidisciplinary team MG Myasthenia gravis MMSE Mini-mental state examination MND Motor neurone disease MRI Magnetic resonance imaging MS Multiple sclerosis MUSK Muscle-specific kinase NCS Nerve conduction study NMDA N-methyl-D-aspartate NMO Neuromyelitis optica NOAC Novel oral anticoagulant OCBs Oligoclonal bands PCR Polymerase chain reaction PLS Primary lateral sclerosis PML Progressive multifocal leukoencephalopathy PMP22 Peripheral myelin protein 22 gene RIM Radiation-induced meningiomas SDH Subdural hematoma TB Tuberculosis VGKC Voltage-gated potassium channel-complex VGCC Voltage-gated calcium channel VPL Ventral posterolateral nucleus of thalamus WBC White blood cell count

Contents

1 Anticoagulation and Fall . . . . . . . . . . . . . . . . . . . . . 1 Thoughts . . . . . . . . . . . . . . . . . . . . . . . . . . . . . . . . . . . 2 Answers . . . . . . . . . . . . . . . . . . . . . . . . . . . . . . . . . . . 4 Hallmarks . . . . . . . . . . . . . . . . . . . . . . . . . . . . . . . . . . 5 Further Reading . . . . . . . . . . . . . . . . . . . . . . . . . . . . 5 Notes . . . . . . . . . . . . . . . . . . . . . . . . . . . . . . . . . . . . . . 6 2 Pain in the Face . . . . . . . . . . . . . . . . . . . . . . . . . . . . .   7 Thoughts . . . . . . . . . . . . . . . . . . . . . . . . . . . . . . . . . . .    8 Answers . . . . . . . . . . . . . . . . . . . . . . . . . . . . . . . . . . .   10 Hallmarks . . . . . . . . . . . . . . . . . . . . . . . . . . . . . . . . . .   11 Further Reading . . . . . . . . . . . . . . . . . . . . . . . . . . . .   12 Notes . . . . . . . . . . . . . . . . . . . . . . . . . . . . . . . . . . . . . . 12 3 Dementia and “Something Else” . . . . . . . . . . . . . . 13 Thoughts . . . . . . . . . . . . . . . . . . . . . . . . . . . . . . . . . . . 15 Further Thoughts . . . . . . . . . . . . . . . . . . . . . . . . . . . . 16 Answers . . . . . . . . . . . . . . . . . . . . . . . . . . . . . . . . . . . 18 Hallmarks . . . . . . . . . . . . . . . . . . . . . . . . . . . . . . . . . . 19 Further Reading . . . . . . . . . . . . . . . . . . . . . . . . . . . . 19 Notes . . . . . . . . . . . . . . . . . . . . . . . . . . . . . . . . . . . . . . 20 4 New Symptoms in Old Illness . . . . . . . . . . . . . . . . . 21 Thoughts . . . . . . . . . . . . . . . . . . . . . . . . . . . . . . . . . . . 22 Answers . . . . . . . . . . . . . . . . . . . . . . . . . . . . . . . . . . . 25 Hallmarks . . . . . . . . . . . . . . . . . . . . . . . . . . . . . . . . . . 25 Further Reading . . . . . . . . . . . . . . . . . . . . . . . . . . . . 26 Notes . . . . . . . . . . . . . . . . . . . . . . . . . . . . . . . . . . . . . . 26 xix

xx

Contents

5 Young Lady with Severe Arm Pain . . . . . . . . . . . . . 27 Thoughts . . . . . . . . . . . . . . . . . . . . . . . . . . . . . . . . . . . 28 Answers . . . . . . . . . . . . . . . . . . . . . . . . . . . . . . . . . . . 30 Hallmarks . . . . . . . . . . . . . . . . . . . . . . . . . . . . . . . . . . 30 Further Reading . . . . . . . . . . . . . . . . . . . . . . . . . . . . 31 Notes . . . . . . . . . . . . . . . . . . . . . . . . . . . . . . . . . . . . . . 31 6 “Remember Bingo Numbers!” . . . . . . . . . . . . . . . . 33 Thoughts . . . . . . . . . . . . . . . . . . . . . . . . . . . . . . . . . . . 34 Answers . . . . . . . . . . . . . . . . . . . . . . . . . . . . . . . . . . . 36 Hallmarks . . . . . . . . . . . . . . . . . . . . . . . . . . . . . . . . . . 37 Further Reading . . . . . . . . . . . . . . . . . . . . . . . . . . . . 37 Notes . . . . . . . . . . . . . . . . . . . . . . . . . . . . . . . . . . . . . . 38 7 Poor Hearing and Seizures . . . . . . . . . . . . . . . . . . . 39 Thoughts . . . . . . . . . . . . . . . . . . . . . . . . . . . . . . . . . . . 40 Answers . . . . . . . . . . . . . . . . . . . . . . . . . . . . . . . . . . . 44 Hallmarks . . . . . . . . . . . . . . . . . . . . . . . . . . . . . . . . . . 44 Further Reading . . . . . . . . . . . . . . . . . . . . . . . . . . . . 45 Notes . . . . . . . . . . . . . . . . . . . . . . . . . . . . . . . . . . . . . . 46 8 Tired and Sleepy Woman with Headaches . . . . . . . 47 Thoughts . . . . . . . . . . . . . . . . . . . . . . . . . . . . . . . . . . . 48 Answers . . . . . . . . . . . . . . . . . . . . . . . . . . . . . . . . . . . 50 Hallmarks . . . . . . . . . . . . . . . . . . . . . . . . . . . . . . . . . . 50 Further Reading . . . . . . . . . . . . . . . . . . . . . . . . . . . . 51 Notes . . . . . . . . . . . . . . . . . . . . . . . . . . . . . . . . . . . . . . 51 9 Disorientation, Confusion I . . . . . . . . . . . . . . . . . . . 53 Thoughts . . . . . . . . . . . . . . . . . . . . . . . . . . . . . . . . . . . 54 Answers . . . . . . . . . . . . . . . . . . . . . . . . . . . . . . . . . . . 57 Hallmarks . . . . . . . . . . . . . . . . . . . . . . . . . . . . . . . . . . 57 Further Reading . . . . . . . . . . . . . . . . . . . . . . . . . . . . 58 Notes . . . . . . . . . . . . . . . . . . . . . . . . . . . . . . . . . . . . . . 58 10 Disorientation, Confusion II. . . . . . . . . . . . . . . . . . . 59 Thoughts . . . . . . . . . . . . . . . . . . . . . . . . . . . . . . . . . . . 60 Answers . . . . . . . . . . . . . . . . . . . . . . . . . . . . . . . . . . . 62 Hallmarks . . . . . . . . . . . . . . . . . . . . . . . . . . . . . . . . . . 63 Further Reading . . . . . . . . . . . . . . . . . . . . . . . . . . . . 63 Notes . . . . . . . . . . . . . . . . . . . . . . . . . . . . . . . . . . . . . . 64

Contents

xxi

11 Woman with Sudden Headache . . . . . . . . . . . . . . . . 65 Thoughts . . . . . . . . . . . . . . . . . . . . . . . . . . . . . . . . . . . 66 Answers . . . . . . . . . . . . . . . . . . . . . . . . . . . . . . . . . . . 69 Hallmarks . . . . . . . . . . . . . . . . . . . . . . . . . . . . . . . . . . 69 Further Reading . . . . . . . . . . . . . . . . . . . . . . . . . . . . 70 Notes . . . . . . . . . . . . . . . . . . . . . . . . . . . . . . . . . . . . . . 71 12

First Seizure. . . . . . . . . . . . . . . . . . . . . . . . . . . . . . . . . 73 Thoughts . . . . . . . . . . . . . . . . . . . . . . . . . . . . . . . . . . . 74 Answers . . . . . . . . . . . . . . . . . . . . . . . . . . . . . . . . . . . 77 Hallmarks . . . . . . . . . . . . . . . . . . . . . . . . . . . . . . . . . . 77 Further Reading . . . . . . . . . . . . . . . . . . . . . . . . . . . . 78 Notes . . . . . . . . . . . . . . . . . . . . . . . . . . . . . . . . . . . . . . 79

13 Elephant in the Room vs. Red Herring. . . . . . . . . . 81 Thoughts . . . . . . . . . . . . . . . . . . . . . . . . . . . . . . . . . . . 82 Answers . . . . . . . . . . . . . . . . . . . . . . . . . . . . . . . . . . . 86 Second Thoughts . . . . . . . . . . . . . . . . . . . . . . . . . . . . 87 Hallmarks . . . . . . . . . . . . . . . . . . . . . . . . . . . . . . . . . . 88 Further Reading . . . . . . . . . . . . . . . . . . . . . . . . . . . . 88 Notes . . . . . . . . . . . . . . . . . . . . . . . . . . . . . . . . . . . . . . 89 14 Gait Disorder and Falls . . . . . . . . . . . . . . . . . . . . . . . 91 Thoughts . . . . . . . . . . . . . . . . . . . . . . . . . . . . . . . . . . . 92 Second Visit . . . . . . . . . . . . . . . . . . . . . . . . . . . . . . . . 92 Second Thoughts? . . . . . . . . . . . . . . . . . . . . . . . . . . . 93 Answers . . . . . . . . . . . . . . . . . . . . . . . . . . . . . . . . . . . 95 Hallmarks . . . . . . . . . . . . . . . . . . . . . . . . . . . . . . . . . . 96 Further Reading . . . . . . . . . . . . . . . . . . . . . . . . . . . . 96 Notes . . . . . . . . . . . . . . . . . . . . . . . . . . . . . . . . . . . . . . 97 15 “Tongue Tied” Man with Opiate Addiction. . . . . . 99 Thoughts . . . . . . . . . . . . . . . . . . . . . . . . . . . . . . . . . . . 100 Answers . . . . . . . . . . . . . . . . . . . . . . . . . . . . . . . . . . . 103 Hallmarks . . . . . . . . . . . . . . . . . . . . . . . . . . . . . . . . . . 104 Further Reading . . . . . . . . . . . . . . . . . . . . . . . . . . . . 105 Notes . . . . . . . . . . . . . . . . . . . . . . . . . . . . . . . . . . . . . . 105 16 Right Sided Hemiparesis in Young Smoker. . . . . . 107 Thoughts . . . . . . . . . . . . . . . . . . . . . . . . . . . . . . . . . . . 108 Answers . . . . . . . . . . . . . . . . . . . . . . . . . . . . . . . . . . . 110

xxii

Contents

Hallmarks . . . . . . . . . . . . . . . . . . . . . . . . . . . . . . . . . . 111 Further Reading . . . . . . . . . . . . . . . . . . . . . . . . . . . . 111 Notes . . . . . . . . . . . . . . . . . . . . . . . . . . . . . . . . . . . . . . 112 17 Young Man with Pins and Needles. . . . . . . . . . . . . . 113 Thoughts . . . . . . . . . . . . . . . . . . . . . . . . . . . . . . . . . . . 114 Answers . . . . . . . . . . . . . . . . . . . . . . . . . . . . . . . . . . . 116 Hallmarks . . . . . . . . . . . . . . . . . . . . . . . . . . . . . . . . . . 117 Further Reading . . . . . . . . . . . . . . . . . . . . . . . . . . . . 117 Notes . . . . . . . . . . . . . . . . . . . . . . . . . . . . . . . . . . . . . . 118 18 Painful and Weak Young Man. . . . . . . . . . . . . . . . . . 119 Thoughts . . . . . . . . . . . . . . . . . . . . . . . . . . . . . . . . . . . 120 Answers . . . . . . . . . . . . . . . . . . . . . . . . . . . . . . . . . . . 122 Hallmarks . . . . . . . . . . . . . . . . . . . . . . . . . . . . . . . . . . 122 Further Reading . . . . . . . . . . . . . . . . . . . . . . . . . . . . 123 Notes . . . . . . . . . . . . . . . . . . . . . . . . . . . . . . . . . . . . . . 123 19 Recurrent Otitis Media. . . . . . . . . . . . . . . . . . . . . . . . 125 Thoughts . . . . . . . . . . . . . . . . . . . . . . . . . . . . . . . . . . . 126 Answers . . . . . . . . . . . . . . . . . . . . . . . . . . . . . . . . . . . 129 Hallmarks . . . . . . . . . . . . . . . . . . . . . . . . . . . . . . . . . . 129 Further Reading . . . . . . . . . . . . . . . . . . . . . . . . . . . . 130 Notes . . . . . . . . . . . . . . . . . . . . . . . . . . . . . . . . . . . . . . 131 20 Deafness and Shoulder Weakness. . . . . . . . . . . . . . . 133 Thoughts . . . . . . . . . . . . . . . . . . . . . . . . . . . . . . . . . . . 134 Answers . . . . . . . . . . . . . . . . . . . . . . . . . . . . . . . . . . . 137 Hallmarks . . . . . . . . . . . . . . . . . . . . . . . . . . . . . . . . . . 137 Further Reading . . . . . . . . . . . . . . . . . . . . . . . . . . . . 138 Notes . . . . . . . . . . . . . . . . . . . . . . . . . . . . . . . . . . . . . . 139 21 Young Girl with the Wrist Drop. . . . . . . . . . . . . . . . 141 Thoughts . . . . . . . . . . . . . . . . . . . . . . . . . . . . . . . . . . . 142 Answers . . . . . . . . . . . . . . . . . . . . . . . . . . . . . . . . . . . 144 Hallmarks . . . . . . . . . . . . . . . . . . . . . . . . . . . . . . . . . . 144 Further Reading . . . . . . . . . . . . . . . . . . . . . . . . . . . . 145 Notes . . . . . . . . . . . . . . . . . . . . . . . . . . . . . . . . . . . . . . 145

Contents

xxiii

22 Facial Tingling and Poor Hearing. . . . . . . . . . . . . . . 147 Thoughts . . . . . . . . . . . . . . . . . . . . . . . . . . . . . . . . . . . 148 Answers . . . . . . . . . . . . . . . . . . . . . . . . . . . . . . . . . . . 151 Hallmarks . . . . . . . . . . . . . . . . . . . . . . . . . . . . . . . . . . 152 Further Reading . . . . . . . . . . . . . . . . . . . . . . . . . . . . 152 Notes . . . . . . . . . . . . . . . . . . . . . . . . . . . . . . . . . . . . . . 153 23 Balance and Gait Problems. . . . . . . . . . . . . . . . . . . . 155 Thoughts . . . . . . . . . . . . . . . . . . . . . . . . . . . . . . . . . . . 156 Answers . . . . . . . . . . . . . . . . . . . . . . . . . . . . . . . . . . . 159 Hallmarks . . . . . . . . . . . . . . . . . . . . . . . . . . . . . . . . . . 160 Further Reading . . . . . . . . . . . . . . . . . . . . . . . . . . . . 160 Notes . . . . . . . . . . . . . . . . . . . . . . . . . . . . . . . . . . . . . . 160 24 Numb Legs and Constipation . . . . . . . . . . . . . . . . . . 161 Thoughts . . . . . . . . . . . . . . . . . . . . . . . . . . . . . . . . . . . 162 Answers . . . . . . . . . . . . . . . . . . . . . . . . . . . . . . . . . . . 164 Hallmarks . . . . . . . . . . . . . . . . . . . . . . . . . . . . . . . . . . 165 Further Reading . . . . . . . . . . . . . . . . . . . . . . . . . . . . 165 Notes . . . . . . . . . . . . . . . . . . . . . . . . . . . . . . . . . . . . . . 165 25 Yellow and Weak Young Man. . . . . . . . . . . . . . . . . . 167 Thoughts . . . . . . . . . . . . . . . . . . . . . . . . . . . . . . . . . . . 168 Answers . . . . . . . . . . . . . . . . . . . . . . . . . . . . . . . . . . . 169 Hallmarks . . . . . . . . . . . . . . . . . . . . . . . . . . . . . . . . . . 169 Further Reading . . . . . . . . . . . . . . . . . . . . . . . . . . . . 170 Notes . . . . . . . . . . . . . . . . . . . . . . . . . . . . . . . . . . . . . . 170 Key Images/Findings . . . . . . . . . . . . . . . . . . . . . . . . . . . . . . 171 Index . . . . . . . . . . . . . . . . . . . . . . . . . . . . . . . . . . . . . . . . . . . 185

About the Author

Ondrej  Dolezal  studied medicine at Charles University in Prague, where he also finished his medical training. He focuses on multiples sclerosis, autoimmunity, epilepsy, neuroimaging and medical education and was also involved in charitable work (medical volunteer in Zambia). He was awarded Dr. Lawrence D. Jacobs Fellowship (Buffalo, USA). After finishing his PhD in neuroscience with a thesis “Grey matter pathology in multiple sclerosis”, he left a tertiary university hospital and started to work in secondary care as a neurology consultant in Dumfries and Galloway, NHS Scotland. He is a fellow of the Royal College of Physicians in Edinburgh.

xxv

Chapter 1 Anticoagulation and Fall

59-year-old gentleman on warfarin for paroxysmal atrial fibrillation, COPD and hypertension presented 2 days after a fall; he was walking his dog, animal pulled the lead and he fell forward. He was brought to the hospital by wife and daughter with forehead bruising and pyrexia (38 °C). Neurological examination at Accident and Emergency was concluded as normal and the same was valid for his cognitive performance. Biochemistry was normal, except INR 3.2 and CRP 25. Patient was started on treatment for urinary infection (positive urine analysis – positive protein). Brain CT scan was requested and a 12  mm hyperdense focus in the left frontal lobe with mild perifocal oedema in keeping with intraparenchymal bleeding was found. It was decided to treat him conservatively. Patient was admitted for antibiotic treatment and observation. During admission patient started to complain about poor balance and dizziness and walking difficulties. In following 48 hours patient developed urinary symptoms (delayed start/retention). On objective examination (see diagram) we found leg hyperreflexia, positive plantars (previously negative). There was relatively mild bilateral leg weakness which was concluded as upper motor neurone paraparesis. There was no convincing sensory level. Cranial nerves (CN) and cerebellar functions were normal.

© Springer Nature Switzerland AG 2019 O. Dolezal, Clinical Cases in Neurology, In Clinical Practice, https://doi.org/10.1007/978-3-030-16628-1_1

1

2

Chapter 1.  Anticoagulation and Fall CN, cerebellar normal

+

+

5/5

5/5

+ +

+ +

+ + Pos. 4+/5

+ + Pos. 4+/5

Thoughts Questions we have to answer in this particular scenario are “Can frontal and very small (12 mm) bleeding cause these new symptoms (sphincters, paraparesis)?” and “Is urinary or other (e.g. neuroinfection) infection playing a role?” Another issue in this case was anticoagulation. While anticoagulation (warfarin and novel oral anticoagulation (NOAC)) is hugely beneficial and literally saves lives, it can be detrimental during surgeries or as above in injuries. Every patient on anticoagulation (or with abnormal INR) should be cautiously examined clinically and more importantly radiologically. Neurologists, neurosurgeons and surgeons tend to develop what can be called “warfarin anxiety”. Especially subdural haematoma, alongside other bleeds, is well recognized complication in the context of injury. This “warfarin anxiety” is never

Thoughts

3

resisted for long by radiologists and CT brain is commonly performed sometimes even in almost asymptomatic anticoagulated individuals after injury (this was the case here as well and this explains early admission CT scan). But back to our patient; it would be rather unlikely if not impossible that intracranial lesion of this extent would be responsible for his new symptoms, which sound rather spinal. His new paraparesis with upper motor neurone features (increased reflexes, positive plantars) and sphincter problems can be explained by spinal cord damage/injury. Another option here would be bilateral supratentorial lesion (e.g. above mentioned subdural haematoma, with rather delayed onset) but initial brain scan was performed already more than 48 hours after the fall so it would be less likely. Neuroinfection is an interesting option here and lumbar puncture would be test of choice, but abnormal INR can lead to complications during and after the puncture. So we should prioritise examinations likely in this order: (1) Cervical and thoracic spine scan, (2) to repeat brain imaging, (3) lumbar puncture (INR reversal/switch to low molecular weight heparin (LMWH) would be needed).

Figure 1.1  MRI sagittal T2 weighted image

4

Chapter 1.  Anticoagulation and Fall

Figure 1.2  MRI sagittal T2 weighted image 2 months later

Key Symptoms Paraparesis (upper motor neurone), urinary retention

Answers After discussing the case it was agreed on MRI of cervical and thoracic spine and as you can see it was very good choice (see Fig.  1.1). There was epidural hematoma at C1/2 level with cord compression and myelopathy (increase cervical cord signal). Initial report mentioned also possibility of subdural location of spinal bleeding. These findings fully explained clinical symptoms (paraparesis, urinary retention) so further tests (LP) were not indicated. Radiologist also mentioned suspected small spinal malformation/arteriovenous malformation at C1-C2 level but subsequent classical angiogram did not reveal any arteriovenous malformation

Further Reading

5

or any other atypical vascularisation. Conservative treatment was r­ecommended by neurosurgeons and patient was temporarily on low molecular weight heparin. Patient gradually improved and recovered fully within 4  weeks (as his MRI 2 months later, see below Fig. 1.2). He is still anticoagulated without residual disability (still walking his dog daily, but more cautiously). Keywords  Myelopathy · Epidural hematoma · Anticoagulation

Hallmarks This case provides interesting mix between very much anticipated complication of blunt injury in somebody who is on anticoagulation  – bleeding/haemorrhage- and unusual location of the bleeding. As clinical symptoms appeared after more than 48 hours after injury we can assume that bleed was slowly progressive, rather venous. This case demonstrated also well known fact that in cranial injuries spinal imaging, namely of cervical spine should be considered.

Further Reading Lindsay KW, Bone I, Fuller G.  Neurology and neurosurgery illustrated e-book. St Louis: Elsevier Health Sciences; 2010. Nadgir R, Yousem DM. Neuroradiology: the requisites. Philadelphia: Elsevier Health Sciences; 2016. Nakanishi K, Nakano N, Uchiyama T, Kato A. Hemiparesis caused by cervical spontaneous spinal epidural hematoma: a report of 3 cases. Adv Orthop. 2011;2011:516382. Park YJ, Kim SW, Ju CIL, Wang HS. Spontaneous resolution of non-­ traumatic cervical spinal subdural hematoma presenting acute hemiparesis: a case report. Korean J Spine. 2012;9:257. Ropper A, Samuels M, Klein J.  Adams and Victor’s principles of neurology. 10th ed. New  York: McGraw Hill Professional; 2014. Scott M.  Spontaneous nontraumatic subdural hematomas. J Am Med Assoc. 1949;141:596–602.

6

Chapter 1.  Anticoagulation and Fall

Notes

Chapter 2 Pain in the Face

32-year-old woman was referred by general practitioner for 3  weeks of gradually progressing left sided facial pain. Pain was present in her forehead, cheek and jaw and probably crossing the facial midline. She had a history of paranoia (schizoaffective disorder mentioned in historical notes) and dependent personality disorder. Patient records also revealed recurrent admissions for minor injuries occurring under influence of alcohol. During examination patient was agitated and it proved to be difficult to examine her properly. She was also very hesitant to let doctors examine her face because of facial pain. However there was allodynia/painful reaction in the left trigeminal territory (all three divisions, seen as black on diagram). Angle of mandible was not painful. Generally motor, cerebellar and sphincter functions were normal, including rest of cranial nerves. There was no alteration of extraocular movements (EOMs). There was hyperreflexia in lower extremities, but normal muscle strength with negative plantars. Rest of her sensory functions were normal.

© Springer Nature Switzerland AG 2019 O. Dolezal, Clinical Cases in Neurology, In Clinical Practice, https://doi.org/10.1007/978-3-030-16628-1_2

7

8

Chapter 2.  Pain in the Face CN, cerebellar normal

+

+

5/5

5/5

+ +

+ +

+ + Neg. 5/5

+ + Neg. 5/5

Thoughts Question in this particular case is “Where, from anatomic point of view, can be a problem?” and “What further tests and potential treatments you would recommend?” Clinical diagnosis based on objective findings was concluded as left sided trigeminal neuralgia (involving ophthalmic, maxillary and mandibular nerves). Knowing patient’s past history is crucial; however it can lead us to wrong assumptions and influence differential diagnostic thoughts. In the light of her colourful psychiatric history and as there was not involved particular division of trigeminal nerve (even anatomical distribution is questionable – crossing somehow the facial midline) we could conclude her case as functional, non organic pain. Also absence

Thoughts

9

of other symptoms would be suggestive of this and almost normal neurological examination could lead our thoughts in that direction. If her symptoms are organic everything is pointing towards the brainstem. Neurovascular ­conflict could be a possibility. Other options such as infection, neoplastic process and vascular origin (aneurysm, bleed) seemed unlikely (duration, otherwise healthy individual, no risk factors etc.). In young women inflammation/demyelination should always be considered. Lumbar puncture was discussed with patient (would be helpful to identify bleed, infection, demyelination) but patient declined. MRI scan to rule out neurovascular conflict or demyelination was requested. Unfortunately patient also declined proposed MRI scan (which was not particularly surprising). She was discharged on symptomatic treatment (Carbamazepine). Patient re-­ appeared back at A&E 2 weeks later as treatment was not helping. There was not any new focal neurology and objective examination was the same as previously. This time patient agreed on MRI with outlook for lumbar puncture.

Figure 2.1  MRI axial FLAIR image

10

Chapter 2.  Pain in the Face

Figure 2.2  MRI axial FLAIR image

Key Symptoms Trigeminal neuralgia

Answers Neurovascular conflict was not present but we found hyperintense lesions in infratentorial and supratentorial regions as well. These changes were suggestive of demyelination/multiple sclerosis (see Figs.  2.1 and 2.2). One of her inflammatory lesions in pons was very close to left trigeminal nerve entry zone and therefore could be primarily responsible for presenting symptoms (see Fig. 2.2). We discussed further options with patient. In multiple sclerosis diagnosis is not only based on MRI picture but clinical picture and cerebrospinal fluid (CSF) positivity are essential. When we discussed our diagnostic thoughts with the patient she admitted having some visual problems (“painful eye with foggy vision”, she could not remember side) at the age of 26, spontaneously recovering

Hallmarks

11

after 3  weeks. This information provided another piece of puzzle as we can assume that this first episode represented optic neuritis. This makes her current sensory episode her second relapse. Lumbar puncture is sometimes sidelined investigation in multiple sclerosis diagnosis but newest Multiple Sclerosis criteria (see further reading below) put more accent on this test again, allowing us to accelerate diagnostic process. Her lumbar puncture found 10 white blood cells/lymphocytes (oligocytosis) and positive oligoclonal bands (OCBs). Clinically she responded almost miraculously to high dose steroids and eventually was started on disease modifying treatment (DMT). Keywords  Trigeminal neuralgia · Multiple sclerosis

Hallmarks This case demonstrated importance of good knowledge of personal medical history. Being aware of patient’s alleged past optic neuritis would likely shift diagnostic thoughts the right direction already during the first encounter. Also information about her pain marginally crossing facial midline is not going against organic origin of patient’s symptoms (anatomically possible) and also uninvolved angle of mandible would be suggestive of organic origin as well (angle of mandible is innervated by cervical roots). Trigeminal neuralgia/facial pain in young women should always be suspicious of demyelination and can be seen in 1.5% of MS patients (as you can read in Neurology by Mark Mumenthaler, Harrison’s manual of medicine and Adams and Victor’s principles of neurology, see below). In clinical practice we very often put inappropriate emphasis and accent on previous psychiatric or addiction ­history. Functional/dissociative symptoms are more and more important part of day-to–day neurological and general practice (to read more about functional disorders you can visit www.neurosymptoms.org). However bottom-line information in this case is that we should strongly suspect organic process in the context of personal medical history and not to be shifted by other co-morbidities especially psychiatric ones.

12

Chapter 2.  Pain in the Face

Further Reading DeMyer W. Technique of the neurologic examination. 5th ed. New York: McGraw Hill Professional; 2003. Harris W. Rare forms of paroxysmal trigeminal neuralgia, and their relation to disseminated sclerosis. Br Med J. 1950;2:1015–9. Kasper DL, Fauci AS, Hauser S, Longo D, Larry Jameson J, Loscalzo J.  Harrisons manual of medicine. 19th ed. New  York: McGraw Hill Professional; 2016. Mumenthaler M, Mattle H.  Neurology. Stuttgart: Thieme; 2011. ISBN 3-13-523904-7. Nadgir R, Yousem DM. Neuroradiology: the requisites. Philadelphia: Elsevier Health Sciences; 2016. ISBN 978-0-323-00508-1. Ropper A, Samuels M, Klein J.  Adams and Victor’s principles of neurology. 10th ed. New York: McGraw Hill Professional; 2014. Thompson AJ, Banwell BL, Barkhof F, et al. Diagnosis of multiple sclerosis: 2017 revisions of the McDonald criteria. Lancet Neurol. 2018;17:162–73. www.neurosymptoms.org.

Notes

Chapter 3 Dementia and “Something Else”

72-year-old gentleman, previously relatively fit, former soldier was admitted for 1 week history of poor balance, instability and gait difficulties. He also mentioned intermittent pins and needles in his legs. There was a past history of chronic lumbar spine issues (including surgery/laminectomy L4, L5 in the past) and longstanding foot drop on the left. He said that “his legs can not bear him” now. He was seen 6 weeks before current admission by psychiatrist with conclusion of mild cognitive decline, possibly related to early Alzheimer’s dementia and he was started on Donepezil. He denied any bladder symptoms only stating that he is not “quick enough to get to toilet in time” because of his leg weakness. Psychiatrist requested CT scan of his brain which was reported normal (6 weeks before current admission). On objective examination there was paraparesis with absent reflexes so seemingly lower motor neurone in origin. His bilateral leg weakness was more prevalent on the left (however we knew about longstanding foot drop – 1/5 muscle strength on diagram below). Plantars were negative. There was reduced proprioception, pinprick and vibration sense in lower extremities and some paresthesias (pins and needles). Patient was admitted under orthopaedics (previous lumbar

© Springer Nature Switzerland AG 2019 O. Dolezal, Clinical Cases in Neurology, In Clinical Practice, https://doi.org/10.1007/978-3-030-16628-1_3

13

14

Chapter 3.  Dementia and “Something Else”

spine problems). During his first 3 days on ward he developed new pins and needles in his right arm but not associated with any weakness. Since admission he was slightly slower cognitively (taking longer to answer at times) but family was certain that this has been the case for last year or two (it was, after all, primary reason for psychiatric review 6 weeks ago). Fundoscopy was normal. CN, cerebellar normal

+

+

5/5

5/5





– Neg. 3/5

– Neg. 1/5

Thoughts

15

Thoughts Question here is what can be unifying diagnosis in this gentleman? Is it only one diagnosis or are there present two separate issues? If we look at general facts it seems that we have patient here with peripheral (lower motor neurone) paraparesis (weakness and absent reflexes), seemingly without sphincter symptoms. Symptoms seems to be progressive  – ­initially symmetrical or with left sided prevalence (likely overlay of previous deficit) but after 72 hours becoming more prevalent on the right (new right arm paresthesias). Therefore his symptoms seemed ascending in nature. That is why demyelinating polyneuropathy (Guillain-Barré Syndrome/GBS) was assumed as the most likely diagnosis. Nerve conduction studies and CSF analysis would be essential for diagnosis, but patient was admitted to secondary hospital with no available electrophysiology service and patient was not keen to be transported. Lumbar puncture was declined very firmly by patient (“already had enough back problems”). There were still some other diagnostic possibilities, for example initial stages of cord compression (spinal shock) can have lower motor neurone features and the same is valid for cauda equina compression. This seemed to be plausible option, but it is true that arm symptoms can not be explained by those pathologies and absent sphincter issues would go against this as well. MRI of whole spine was arranged. It found only widespread and severe degenerative changes with multiple level root compressions (especially L5 on the left). MRI seemed to be stable when compared with previous MRI performed 2 years ago. As symptoms were progressing treating doctors agreed on possible diagnosis of autoimmune inflammatory polyradiculoneuropathy (GBS) and we started empirical treatment with intravenous immunoglobulin (IVIG). After further 3  days patient was not clinically improving but reached “plateau” and his condition remained stable. During neurological review 5 days later (see the second diagram below) there was quite convincing episode of incontinence (patient not being even aware) and further cognitive slowing according to his family (did not recognise his daughter during recent visit).

16

Chapter 3.  Dementia and “Something Else” CN, cerebellar normal

5/5

5/5

+

+

5/5

5/5





– Neg. 3/5

– Neg. 1/5

Further Thoughts Bladder problems would be very unlikely in inflammatory polyneuropathy and we did not have any evidence about cord compression (MRI spine was normal). Sphincter issues therefore should again lead us to rather central cause of his problems. Progressive cognitive issues were also unexpected (despite could be explained by hospitalisation, medication, lack of sleep etc.). Therefore MRI of brain was arranged (CT performed as above before admission) with outlook of performing lumbar puncture.

Further Thoughts

Figure 3.1  MRI coronal FLAIR image

Figure 3.2  MRI axial FLAIR image

17

18

Chapter 3.  Dementia and “Something Else”

Figure 3.3  CT brain axial image

Key Symptoms Incontinence, cognitive decline, gait disorder

Answers As you can see bilateral subdural subacute/chronic ­haematomas were found more prominent on the left (Figs. 3.1 and 3.2) with some “fresh bleed” on the left (see Fig.  3.2). These findings explained all patients’ symptoms. Also signs of more recent bleeding on the left would fit with new onset of right arm paresthesias during admission. Patient had urgent surgery (see scan below) and improved markedly (from motor and cognitive point of view as well) (Fig. 3.3). Keywords  Subdural haematoma · Guillain-Barré syndrome · Radiculopathy · Incontinence

 

Further Reading

19

Hallmarks This perplexing case demonstrated couple of obvious errors or rather misjudgements, despite clinicians relied on the clinical picture. Patient’s symptoms seemed to originate from lower back/spine in general and central lesion was not suspected and doctors were reassured by pre-admission brain scan which was done 6 weeks before admission. We assumed intracranial findings to be normal and cognitive decline was believed to be related to previous diagnosis of dementia. Probably most confusing was that clinical picture was undeniably obscured by chronic compressive polyradiculopathy/ root compression. Therefore paraparesis (retrospectively obvious upper motor neurone in origin) seemed falsely lower motor neurone in nature. Ascending nature of his symptoms led to further confusion. Lack of head injury in personal medical history was also misleading here (but it is usually the case in chronic/subacute subdural haematomas (SDH)). Potentially very dangerous would be performing lumbar puncture in similar setting (without new brain scan) with very high risk of herniation of the brain.

Further Reading Lindsay KW, Bone I, Fuller G.  Neurology and neurosurgery illustrated e-book. St.Louis: Elsevier Health Sciences; 2010. Nadgir R, Yousem DM. Neuroradiology: the requisites. Philadephia: Elsevier Health Sciences; 2016. van den Doel EM. Balzac’s “Pierette”.An early description of chronic subdural hematoma. Arch Neurol. 1986;43:1291–2.

20

Chapter 3.  Dementia and “Something Else”

Notes

Chapter 4 New Symptoms in Old Illness

75-year-old man with ankylosing spondylitis (Bechterew’s disease) was admitted with walking difficulties. Symptoms had gradually progressed over the period of 2–3 years. There was bilateral leg weakness, with left leg being described as “less reliable” than the right. Patient had suffered from ankylosing spondylitis for decades and for last 2  years he had been off all treatments. In the past whole spine “irradiation” was used (no further information had been available, historical notes also not available). On objective examination we found paraparesis/weakness with prevalence on the left (4/5 vs. 4+/5 on the right). There was hyperreflexia in legs with clonus in left ankle. Patient had obvious sphincter problems (having catheter for more than 3  years for retention- so likely pre-­dating walking issues). There was light touch hypoesthesia in lower extremities and loss of vibration sense, there was not any sensory level on trunk. Plantars were positive/extensor. Findings in upper extremities were normal. Cranial nerves and cerebellar functions were unremarkable as well.

© Springer Nature Switzerland AG 2019 O. Dolezal, Clinical Cases in Neurology, In Clinical Practice, https://doi.org/10.1007/978-3-030-16628-1_4

21

22

Chapter 4.  New Symptoms in Old Illness CN, cerebellar normal

+

+

5/5

5/5

+ +

+ +

+ + Pos. 4+/5

+ + + Pos. 4/5

Thoughts Important question here is about localisation of possible lesion and what would be the most useful test. From neurological examination we know that there is upper motor neurone paraparesis (increased reflexes, extensor plantars, and weakness) and longstanding bladder problems (likely retention), therefore spinal cord lesion/compression should be considered. Bilateral supratentorial lesion e.g. subdural haematoma can present in similar fashion (compare with previous case – case 3) but it is less likely. We also know, again thanks to neurological examination, that as there are not any signs in arms/hands so lesion can be somewhere below T1 level. Absence of sensory level on trunk can be surprising but is not always fully developed in cord lesions.

Thoughts

Figure 4.1  MRI sagittal T2 weighted image

Figure 4.2  MRI sagittal T1 weighted image with gadolinium

23

24

Chapter 4.  New Symptoms in Old Illness

Figure 4.3  MRI axial T2 weighted image

Figure 4.4  MRI sagittal T2 weighted image

Key Symptoms Upper motor neurone/central paraparesis, urinary retention

 

Hallmarks

25

Answers MRI of whole spine was arranged. As you can see there were meningiomas, one larger in thoracic (Fig. 4.1) and one smaller in lumbar spine (Fig.  4.4). Thoracic meningioma was almost completely filling spinal canal, compressing spinal cord at T3-T4 level, displacing cord posteriorly (Fig. 4.3). On second picture (Fig. 4.2) you can see gadolinium enhancement of meningioma. Multiple neurofibromas in cauda equina were also found as well (dark “beads” on roots) (Fig. 4.4). Neurofibromas seem to be causing minimal clinical problems and are of limited clinical value in clinical context (coincidental). Patient was treated conservatively and neurosurgery was dismissed (patient not keen, primary diagnosis, respiratory issues probably linked to late stage spondylitis which would interfere with anaesthesia and postoperative reconvalescence etc.). Keywords Ankylosing spondylitis · Radiotherapy · Meningioma · Schwannoma

Hallmarks Interestingly we found multiple lesions but T spine meningioma (largest) seems to be responsible for majority of clinical symptoms  – by compression of urinary, motor and sensory tracts. In similar cases we usually do not have difficulties to identify spinal cord lesion as responsible for symptoms. Cord compression with myelopathy is less frequent in thoracic than in cervical spine. In this case presence of multilevel meningiomas seems surprising. However key information is mentioned in the first paragraph. Meningiomas (and to some extent Schwannomas) can be induced by radiotherapy (radiotherapy induced meningiomas (RIM)). Radiation therapy can have similar consequences in intracranial territory as well (meningiomas and Schwannomas as well, see further reading). Conservative treatment can seem unexpected but ­radical decompression would not lead to improvement and would rather aim for clinical stability. So when we take into

26

Chapter 4.  New Symptoms in Old Illness

account very slowly progressive symptomatology, risks of surgery and most importantly patient’s wishes, conservative “watch and wait” strategy did not seem unreasonable.

Further Reading Al Shalchi M, Hussain S, Giridharan S, Albanese E.  Rare case of radiation-induced trigeminal schwannoma occurring in a long-­ term glioblastoma multiforme survivor. BMJ Case Rep. 2016; https://doi.org/10.1136/bcr-2016-214438. Godlewski B, Drummond KJ, Kaye AH. Radiation-induced meningiomas after high-dose cranial irradiation. J Clin Neurosci. 2012;19:1627–35. Lindsay KW, Bone I, Fuller G.  Neurology and neurosurgery illustrated e-book. St Louis: Elsevier Health Sciences; 2010. Nadgir R, Yousem DM. Neuroradiology: the requisites. Philadelphia: Elsevier Health Sciences; 2016. Rubinstein AB, Reichenthal E, Borohov H.  Radiation-induced schwannomas. Neurosurgery. 1989;24:929–32. Shah AH, Jusue-Torres I, Kuchakulla M, Ivan ME, Benveniste RJ, Morcos JJ, Komotar RJ.  Radiation-induced meningiomas: a case-control study at single center institution. J Neurol Sci. 2018;387:205–9. Yamanaka R, Hayano A, Kanayama T.  Radiation-induced meningiomas: an exhaustive review of the literature. World Neurosurg. 2017;97:635–44.e8.

Notes

Chapter 5 Young Lady with Severe Arm Pain

30-year-old very active, hardworking woman attended our clinic with sharp pain in the left arm and hand. Her left leg “probably felt a bit different too”. On further questioning she mentioned some discomfort in the abdomen on the left as well. Pain was present for 11  months; started as paresthesias (pins and needles) and was gradually progressive. She denied other symptoms including sphincters. She did not respond to neuropathic pain treatments. On objective testing we found monofilament sensation reduced and proprioception and vibration sense was impaired as well in left upper extremity. There was no convincing objective sensory deficit in lower extremities (only subjective as above). Sensory deficit/light touch hypoesthesia was seen in the abdomen and chest with no convincing sensory level. Reflexes were present and symmetrical and there was no paresis. From sensory point of view face was spared and cranial nerves and cerebellar functions were normal.

© Springer Nature Switzerland AG 2019 O. Dolezal, Clinical Cases in Neurology, In Clinical Practice, https://doi.org/10.1007/978-3-030-16628-1_5

27

28

Chapter 5.  Young Lady with Severe Arm Pain CN, cerebellar normal

+

+

5/5

5/5

+

+

+ Neg. 5/5

+ Neg. 5/5

Thoughts In this case we again should ask simple question about location of lesion and decide what other tests should be arranged. This is a case of rather isolated sensory symptoms (sensory hemihypoesthesia, allodynia and pain with spared face (despite sensory deficit in leg was rather subjective). As both upper and lower extremities were involved pathology/lesion could be located in the spinal cord above or at the origin of

Thoughts

29

cervical roots (above C5). As said deficit is primary sensory with spared corticospinal/motor tracts. Supratentorial lesion with involvement of spinothalamic tract and sensory cortex (postcentral gyrus, Brodmann area 1, 2, 3) should be considered as well. Genuine central neuropathic pain is rarer than peripheral type and usually is associated with variable processes involving spinothalamic tract on different levels. Pain and sensory symptoms did not respect any particular dermatome and were asymmetrical also therefore peripheral nerve involvement was unlikely. Because of her age demyelination/ Multiple sclerosis should be considered but neoplasm must not be overlooked. Infectious aetiology seems unlikely in chronic cases like hers (symptoms for almost 1  year). MRI scan of brain and cervical spine was requested. Lumbar puncture was planned to follow.

Figure 5.1  MRI axial FLAIR image

30

Chapter 5.  Young Lady with Severe Arm Pain

Key Symptoms Neuropathic pain, incomplete hemihypoesthesia

Answers MRI showed changes suggestive of demyelination, fulfilling McDonald’s criteria for MS. One of the lesions was found in proximity and involving right lateral thalamus (Fig. 5.1) probably being responsible for central neuropathic pain (despite lesion is slightly away from ventral posterior lateral nucleus – VPL). Lumbar puncture found oligocytosis (25 leukocytes, 100% lymphocytes) and positive oligoclonal bands (OCBs). As this occurred before 2017 updated Multiple Sclerosis (MS) criteria patient was diagnosed as clinically isolated syndrome (CIS) (first symptom of demyelination/multiple sclerosis). She was initially started on high dose steroids (with very good clinical response as the pain settled quickly) and eventually on disease modifying treatment (DMT). Keywords  Neuropathic pain · Multiple sclerosis

Hallmarks This case is not dissimilar to case 2. Other conditions such as chronic pain syndrome, fibromyalgia, compressive cervical radiculopathy in hard working individual etc. were proposed over the period of 12 months. Lack of other alarming symptoms such as motor deficit, sphincter and coordination problems can lead to underestimation of illness severity. Examination of sensory modalities is affected by its subjective and sometimes inconsistent nature. We also heavily rely on patient’s compliance; therefore sensory symptoms can be often underestimated. The key to this case was again the neurological examination.

Notes

31

Further Reading Campbell JN, Meyer RA. Mechanisms of neuropathic pain. Neuron. 2006;52:77–92. DeMyer W. Technique of the neurologic examination. 5th ed. New York: McGraw Hill Professional; 2003. Nadgir R, Yousem DM. Neuroradiology: the requisites. Philadelphia: Elsevier Health Sciences; 2016. Ropper A, Samuels M, Klein J.  Adams and Victor’s principles of neurology. 10th ed. New York: McGraw Hill Professional; 2014.

Notes

Chapter 6 “Remember Bingo Numbers!”

75-year-old woman was referred to us by general practitioner for two episodes of “memory lapses” (3 and 1  month ago respectively). On the first occasion she was sitting at the bingo waiting for numbers to be drawn for the last time that evening. The next thing she remembers is that everyone else was leaving the room as the game was over. She lost approx. 3–5 minutes. She was not communicating with others during that time but her friends assumed that she was focusing on the game. Second episode led to an accident when she drove her mobility scooter into a fence, not knowing why and not having any recollection about it. She did not have any incontinence, bitten tongue or obvious seizure activity. Objective findings were generally normal, with no obvious paresis, sensory or cerebellar signs. Ankle reflexes were absent. She walked unsupported but for longer distances (shopping) she used a mobility scooter. During examination she did have episode when she looked somehow disengaged and “not interested”. She was looking around she did not answer any of questions and she was ignoring instructions. She instead was looking for something in her handbag. This lasted for 20 seconds and afterwards she was not confused and we could continue our discussion.

© Springer Nature Switzerland AG 2019 O. Dolezal, Clinical Cases in Neurology, In Clinical Practice, https://doi.org/10.1007/978-3-030-16628-1_6

33

34

Chapter 6.  “Remember Bingo Numbers!” CN, cerebellar normal

+

+

5/5

5/5

+

+

– Neg. 5/5

– Neg. 5/5

Thoughts Our situation was easier here as we witnessed one episode which was likely similar in origin to previous episodes. The fact that we witnessed this episode in the clinic was also suggestive that these episodes could be more frequent than originally thought. We can probably call her episode loss of

Thoughts

35

awareness with subsequent amnesia. Consciousness was preserved. Patient did not seem to be postictal and there was a lack other seizure markers. Cardiogennic origin (syncope, orthostatic hypotension etc.) we can dismiss here. Epilepsy would be on top of the list with likely complex partial (focal) seizures with automatisms. We can sometimes see term psychomotor seizures in older literature. This presentation would be typical for temporal or frontal seizures. Other option we have to consider would be non epileptic/functional episodes, metabolic causes (intoxication), basilar migraine etc. But as this is the new onset of seizure disorder in elderly patient and seizures can be secondary to intracranial lesion (specifically in temporal or frontal region), we should not hesitate to obtain a brain scan.

Figure 6.1  MRI axial T2 weighted image

36

Chapter 6.  “Remember Bingo Numbers!”

Figure 6.2  MRI axial T2 weighted image

Key Symptoms Loss of awareness/automatisms

Answers MRI showed lesion in right medial fossa with compression of brain tissue and extensive perifocal oedema (Figs.  6.1 and 6.2). After discussion with radiologist it was concluded as likely meningioma with fast growth with reactive oedema of adjacent brain tissue. Antiepileptic treatment was started (lamotrigine) alongside Dexamethasone (antiedema treat-

Further Reading

37

ment) with good response. Follow-up scans were stable after 3 and 6  months respectively. Patient refused surgery at the time so intervention was postponed. Keywords  Focal seizures · Temporal lobe · Meningioma

Hallmarks Clinician here was very lucky as he witnessed seizure which very rarely happens in clinical practice. When we think about epileptic seizures, everyone imagines generalised tonic clonic seizures, but not always considers similar “memory lapses” as being epileptic. As mentioned above differential diagnosis of patient’s episodes would be dissociative non epileptic (psychogenic) seizures. But in similar cases (especially in adults over 65  years of age) and because of recurrence, uniformity and consistency of patient’s episodes functional origin was unlikely and scan was warranted. Type of the tumour is not surprising as meningiomas are more frequent in postmenopausal women.

Further Reading Englot DJ, Chang EF, Vecht CJ. Epilepsy and brain tumors. Handb Clin Neurol. 2016;134:267–85. Johnson DR, Olson JE, Vierkant RA, Hammack JE, Wang AH, Folsom AR, Virnig BA, Cerhan JR.  Risk factors for meningioma in postmenopausal women: results from the Iowa Women’s Health Study. Neuro-Oncology. 2011;13:1011–9. Nadgir R, Yousem DM. Neuroradiology: the requisites. Philadelphia: Elsevier Health Sciences; 2016.

38

Chapter 6.  “Remember Bingo Numbers!”

Notes

Chapter 7 Poor Hearing and Seizures

60-year-old woman was admitted via A&E after ambulance was called by family. She was feeling generally well, only complaining about mild to moderate headaches for couple of days. Headache was more on the left side but was rather global. On the day of admission she lost consciousness without any warning signs, collapsed on the floor and developed generalised whole body jerking. When paramedics entered the scene they still observed generalised tonic clonic seizure. Seizure lasted between 10 and 15 minutes according to family with some “breaks” in seizure activity, when the jerking settled temporarily. She regained consciousness in the ambulance (after receiving Diazepam 10 mg IV and oxygen). On arrival to A&E patient was conscious (GCS 15), and minimally post ictal (some confusion, latency in verbal responses, fatigue). She was still complaining about headache. Patient was admitted to medical ward. Objective findings on admission were almost normal. There was only hypoacusis on the left (not on diagram). Weber test was shifted towards left and Rinne was reduced on the left (so suggestive of conductive hearing loss). Patient was still complaining about headache more on the left (mostly temple and around her ear) and there was still some postictal confusion but this was visibly improving. There were no signs of meningism/meningeal irritation (no nuchal rigidity). She was cognitively back to normal at the time of further neurological examination approximately © Springer Nature Switzerland AG 2019 O. Dolezal, Clinical Cases in Neurology, In Clinical Practice, https://doi.org/10.1007/978-3-030-16628-1_7

39

40

Chapter 7.  Poor Hearing and Seizures

6  hours after admission. There was negative family history of epilepsy. Patient and family denied alcohol or illicit substances involvement. Blood tests showed CRP of 145 and WBC was 15 in peripheral blood. Chest X-ray and urinalysis was normal. CN, cerebellar normal

+

+

5/5

5/5

+

+

– Neg. 5/5

– Neg. 5/5

Thoughts Question we should ask is regarding further management as so far diagnosis of isolated seemingly unprovoked epileptic seizure seemed obvious. Main difficulty we have in similar cases (also in case 6) is to decide if seizures are “primary”

Thoughts

41

(representing disease) or “secondary” to other brain pathology, representing symptom of something else. If seizure is secondary to other pathology (tumour, infection, bleed, trauma) we do not always see other signs of focal neurological deficit (as in the case 6) and objective examination can be normal. Secondary seizures are usually caused by cortical irritation and tend to have focal origin with subsequent generalisation. Very important is that patient in this case was not completely asymptomatic (different to patient 6) as headache (more on the left) persisted which was unusual for the patient. New finding of hypoacusis was also surprising as she was not subjectively complaining about that. Brain scan - CT or MRI – in this case would be the step in the right direction. When we looked at blood results it seemed that infectious aetiology is plausible so lumbar puncture (LP) would be essential as well. Scan will help us to rule out also space occupying lesion (increased intracranial pressure (ICP)) before LP.

Figure 7.1  MRI axial FLAIR image

42

Chapter 7.  Poor Hearing and Seizures

Figure 7.2  MRI axial T2 weighted image

Figure 7.3  MRI axial T2 weighted image

Thoughts

43

a

b

Figure 7.4 MRI axial T1 weighted image without (a) and with Gadolinium contrast (b) respectively

44

Chapter 7.  Poor Hearing and Seizures

Key Symptoms Hypoacusis, seizure

Answers Initially CT of head was performed (not shown here) and found soft tissue density in left mastoid cells which normally should contain air. These changes are typically infectious/ inflammatory. Similar changes were found in ethmoids and left maxillary sinus. There was no obvious collection or obvious inflammatory changes intracranially. There was no space-­ occupying lesion. MRI brain with contrast was requested. T2 weighted images/FLAIR images showed mastoiditis present on the left (Figs. 7.1 and 7.2) and swelling of adjacent left temporal lobe (Fig. 7.3) and T1 weighted images with gadolinium showed leptomeningeal enhancement in left temporo-­parietal region (last two scans: non contrast (Fig.  7.4a) vs. contrast (Fig.  7.4b). Lumbar puncture followed and found 8 WBC (lymphocytes) and protein 0.58  g/l. which was a relatively borderline result. Antibiotics and antivirals were started. Discussion with surgeons followed. After some hesitation we proceeded to mastoid biopsy. Group A streptococcus (sensitive to clarithromycin) was isolated from that. After biopsy antibiotics were modified according to sensitivity. Patient started on prophylactic antiepileptic treatment (Carbamazepine) and did not develop any further seizures. Follow-up MRI scan in 6 months showed complete resolution of changes (not shown). Keywords  Seizures · Hypoacusis · Meningitis · Mastoiditis

Hallmarks History in patients with first seizure should be always taken in detail and possible trigger factors should be identified. If there are no known trigger factors and patient reports other symp-

Further Reading

45

toms being present before seizure onset (new headaches, memory problems, personality changes, hearing issues etc.) or symptoms and signs are present after seizure (paresis, other persistent focal neurology etc.) scanning should always be performed. In this case CT scan led us to correct diagnosis but did not provide sufficient information about situation in central nervous system namely in temporal lobe and about intracranial spread of infection. Association of mastoiditis and meningitis is well known (see below even article from 1916). MRI was crucial in identifying leptomeningeal and parenchymal involvement. Lumbar puncture clearly demonstrated “infection spread” from extradural area (mastoid) intracranially. Meningitis/meningoencephalitis was likely reactive. It is remarkable that in bacterial infection CSF analysis did not show any polymorphs but only lymphocytes, but this could be typical for initial stages of bacterial infection and for reactive inflammation. Interesting is lack of meningeal signs clinically but it does not have to be seen in initial stages of meningitis. Generally findings concurred with diagnosis of infectious bacterial mastoiditis on the left with secondary rather localised meningitis with reactive temporal lobe swelling/ involvement.

Further Reading Huntington WH. Case of mastoiditis complicated by purulent cerebrospinal meningitis; operation and recovery. J Am Med Assoc. 1916;LXVII:201. Lindsay KW, Bone I, Fuller G.  Neurology and neurosurgery illustrated e-book. St. Louis: Elsevier Health Sciences; 2010. Mumenthaler M, Mattle H. Neurology. Stuttgart: Thieme; 2011. Ropper A, Samuels M, Klein J.  Adams and Victor’s principles of neurology. 10th ed. New York: McGraw Hill Professional; 2014.

46

Chapter 7.  Poor Hearing and Seizures

Notes

Chapter 8 Tired and Sleepy Woman with Headaches

48-year-old female was admitted with a 3  days history of “being unwell”, tired and having headaches. Eventually she developed confusion and became progressively sleepy (oscillating level of consciousness (GCS between 14 and 11). According to family she had been incontinent over that period. On admission patient was drowsy, confused and pyrexic (38.1  °C). Her responses were incoherent (usually responded by single words or provided yes/no answers). On objective examination left sided hemiparesis was found, with increased reflexes on the left. There was bilateral ataxia visible especially when walking (gait ataxia) and during finger/nose test (dysmetria). There was also dysdiadochokinesia. There was around 350  ml of residuum in her bladder. There were positive plantars bilaterally. Blood tests revealed CRP of 35 with only borderline leukocytosis.

© Springer Nature Switzerland AG 2019 O. Dolezal, Clinical Cases in Neurology, In Clinical Practice, https://doi.org/10.1007/978-3-030-16628-1_8

47

48

Chapter 8.  Tired and Sleepy Woman with Headaches CN, cerebellar abnormal/ataxia

+ +

+

5/5

3/5

+ +

+

+ Pos. 5/5

+ + Pos. 3/5

Thoughts In this case doctors did not struggle to identify a clearly very unwell lady with deteriorating level of consciousness and focal neurology (left sided upper motor neurone hemiparesis, cerebellar signs etc.). Diagnostic thoughts involved brain bleed (sometimes you can see pyrexia, GCS changes, headaches, focal neurology etc.) but ongoing rather progressive nature would be suggestive of neuroinfection. Viral meningoencephalitis would be on top of the list, so the main issue was to plan further tests and treatment in a timely fashion. As patient clinically progressed it was prudent to start antibiotic and antiviral treatment as soon as possible and without delay. CT brain was requested (to be certain about intracranial expansion/space occupying lesion, hydrocephalus) and lumbar puncture followed.

Thoughts

Figure 8.1  CT brain, axial image

Figure 8.2  CT axial image

49

50

Chapter 8.  Tired and Sleepy Woman with Headaches

Key Symptoms Hemiparesis, change in level of consciousness (GCS)

Answers CT scan of the brain showed hyperdensities in the temporal lobe on the left (adjacent to the hippocampus) and swelling (hyperaemia) (Fig. 8.1). These hyperdensities were suggestive of bleeding concurring with temporal haemorrhagic encephalitis. There were milder but similar changes in the right temporal lobe as well. Three days later there was established reduction in volume within both temporal lobes on the CT (suggestive of necrosis/ encephalomalacia). Lumbar puncture/CSF analysis showed increased protein of 1.2 g/l and 100 WBC (monocytes and lymphocytes). Diagnosis of viral encephalitis was proven at this point. PCR put a rubber stamp on this diagnosis when we obtained positive PCR for Herpes Simplex Virus type 1 (HSV1). Patient survived but was left with treatment resistant secondary generalised epilepsy and significant personality and memory changes. On follow up scan (Fig. 8.2 below) 5 years later significant chronic temporal lobe changes can be seen. Keywords  Viral encephalitis · Temporal lobe · Herpes Simplex Virus

Hallmarks As mentioned above doctors at A&E and receiving physicians usually do not struggle to identify patient at risk in similar situation (“acutely ill patient”). This patient was very unwell following admission and progressive neurological deficit was clearly suggestive of infectious aetiology. Prognosis of herpetic encephalitis is commonly poor. When this does not lead to death (having very high mortality) it causes significant morbidity as in this case. Level of consciousness (GCS) is a very important prognostic factor. As our patient

Notes

51

had already reduced level of consciousness prognosis was poor. Even in early treated patient with no seizures or reduced GCS only around one third of patient heal without clinical consequences or disability. Herpes simplex virus is responsible for around 8% of encephalitis cases.

Further Reading Eran A, Hodes A, Izbudak I. Bilateral temporal lobe disease: looking beyond herpes encephalitis. Insights Imaging. 2016;7:265–74. Hasbun R, Rosenthal N, Balada-Llasat JM, Chung J, Duff S, Bozzette S, Zimmer L, Ginocchio CC.  Epidemiology of meningitis and encephalitis in the United States, 2011–2014. Clin Infect Dis. 2017;65:359–63. McGrath N, Anderson NE, Croxson MC, Powell KF. Herpes simplex encephalitis treated with acyclovir: diagnosis and long term outcome. J Neurol Neurosurg Psychiatry. 1997;63:321–6. Misulis KE, Head TC. Netter’s concise neurology updated edition. Philadelphia: Elsevier Health Sciences; 2016. Mumenthaler M, Mattle H. Neurology. Stuttgart: Thieme; 2011. Ropper A, Samuels M, Klein J.  Adams and Victor’s principles of neurology. 10th ed. New York: McGraw Hill Professional; 2014.

Notes

Chapter 9 Disorientation, Confusion I

47-year-old woman presented with a 2  year history of recurrent events where she suddenly “did not known what she was doing”; for example having the remote control for the television in her hand and not knowing what to do with it. But within 1 minute she was able to use it again. Interestingly she also noticed that she had a funny feeling described as “something was going to happen before it had happened”, or that she can see something which has already happened. Another episode occurred while she was getting juice from the fridge. She couldn’t work out how to actually open and pour it into the glass which was sitting beside the carton of juice. These episodes can take place several times a day, although she can go for several weeks without obvious incidents. Her speech “sometimes went” as well and she could not express herself properly. She was not sure if her ability of understanding is preserved completely during these spells. When admitted she looked very anxious and agitated. She had some word finding difficulties at times on admission. Anxiety was blamed for her symptoms by her and family. On objective examination there was hyperreflexia in all four extremities and plantars were extensor/positive. Cranial nerves and cerebellar functions were normal. Otherwise no neurological pathology was found.

© Springer Nature Switzerland AG 2019 O. Dolezal, Clinical Cases in Neurology, In Clinical Practice, https://doi.org/10.1007/978-3-030-16628-1_9

53

54

Chapter 9.  Disorientation, Confusion I CN, cerebellar normal

+ +

+ +

5/5

5/5

+ +

+ +

+ + Pos. 5/5

+ + Pos. 5/5

Thoughts Essential question was if these episodes represented organic lesion or could be stress related. Being stress related was not completely impossible as there was an element of anxiety present. It is also important to look at her symptoms also from reverse perspective because focal/partial especially temporal epileptic seizures can trigger and can be manifested as ­“anxiety” also (feeling of imminent danger, non-specific fear of “something” going to happen, attempts to escape/run away etc.); Generally features of emotional instability, dyspraxia/apraxia and “déjà vu” episodes would be typical of focal epileptic seizures. As there was speech involved at times (dysphasia/aphasia) we can expect origin to be in the left temporal lobe. Brain

Thoughts

55

i­ maging would be beneficial to rule out focal lesion, especially as there is no previous history of epilepsy. MRI would seem better choice as CT would not pick up gentle changes such as mesial temporal sclerosis and less obvious lesions (smaller meningioma, glioma etc.). As episodes were getting more frequent we requested scan on an urgent basis. EEG was requested as well.

Figure 9.1  MRI axial T2 weighted image

Figure 9.2  MRI axial T1 weighted image with Gadolinium contrast

56

Chapter 9.  Disorientation, Confusion I

Figure 9.3  MRI axial FLAIR image

Figure 9.4  MRI axial T2 weighted image

Key Symptoms Dyspraxia, dysphasia, “déjà vu” episodes

 

Hallmarks

57

Answers MRI was very abnormal. It found a solitary well defined extra axial mass in the left middle cranial fossa anteriorly (Fig.  9.1). On the second picture (Fig.  9.2) you can see an enhancement of the tumour but also of adjacent dura (dural tail). There was severe vasogenic oedema with compression of the left lateral ventricle. There was also midline shift of 1 cm and right lateral ventricle dilation suggestive of partial hydrocephalus (compression of foramen Monro on the contralateral side – on the right) (Figs. 9.3 and 9.4). Findings were in keeping with meningioma. The patient was urgently discussed with neurosurgeons and dexamethasone was started. She was also started on an antiepileptic drug (levetiracetam) with incomplete response. Successful surgery took place (complete resection) and seizures settled gradually. Within 2 years antiepileptic medication was fully discontinued. Keywords  Focal seizures · Temporal lobe · Meningioma

Hallmarks This case demonstrated probably the most feared scenario when neurological signs are interpreted for long time as linked to anxiety or not being organic. Even more confusing is this situation when the explanation is accepted by patient (quite often you hear comments like “I am an anxious ­person.” or “I am a worrier.”). Disturbing symptoms in this particular case seemed to be dyspraxia, dysphasia and “déjà vu” episodes. These neurological symptoms can be easily identified and represent cortical temporal lobe signs. Meningioma is a benign (with exemption of invasive ­malignant meningioma which is rare) neoplasm but in this particular case increased intracranial pressure caused by tumour and oedema almost led to life threatening situation (midline shift, potential herniation etc.)

58

Chapter 9.  Disorientation, Confusion I

Further Reading Englot DJ, Chang EF, Vecht CJ. Epilepsy and brain tumors. Handb Clin Neurol. 2016;134:267–85. Kasper DL, Fauci AS, Hauser S, Longo D, Larry Jameson J, Loscalzo J.  Harrisons manual of medicine. 19th ed. New York: McGraw Hill Professional; 2016. Misulis KE, Head TC. Netter’s concise neurology updated edition. Philadelphia: Elsevier Health Sciences; 2016. Nadgir R, Yousem DM. Neuroradiology: the requisites. Philadelphia: Elsevier Health Sciences; 2016.

Notes

Chapter 10 Disorientation, Confusion II

44-year-old man was reviewed in the neurology outpatient clinic for solitary episode of loss of awareness, possibly consciousness. He had had a very busy day and came back home at around 5 pm. There was an argument in the house which involved his daughter (teenager) and his partner. They were very going on holiday soon and were planning on leaving their daughter at home alone so the heated discussion revolved around that. Surprisingly his daughter developed chest pain during the argument so they got into the car and patient started to drive her to hospital. However when they were still close to home, the patient suddenly stopped responding, but he tightly gripped the steering wheel. The episode lasted between 5 and 10  seconds and led to a low speed road traffic accident (he hit a sign post, airbag was not deployed). According to his partner it seemed that he moved the wheel slightly to the side to avoid a car going opposite direction so it seemed that he did not lose complete control of his actions. The patient mentioned during examination that he could hear what other people were telling him/saying and that “people were screaming in the car”. He was not “himself” and was quite vacant and confused after the accident for 5–10 minutes. His partner said that he was “different” for the rest of the day. Objective neurological findings were normal. with only S1 areflexia. Cranial nerves and cerebellar functions were normal. The patient looked relatively anxious during examination. He credited this episode to stress and the argument. © Springer Nature Switzerland AG 2019 O. Dolezal, Clinical Cases in Neurology, In Clinical Practice, https://doi.org/10.1007/978-3-030-16628-1_10

59

60

Chapter 10.  Disorientation, Confusion II CN, cerebellar normal

+

+

5/5

5/5

+

+

– Neg. 5/5

– Neg. 5/5

Thoughts Clear question here is if this is in fact a neurological problem? Similar situations and episodes are commonly seen at A&E and outpatient Neurology clinics, including stress related simple faints or dissociative episodes. In this particular case we cannot be certain if the whole episode was not stress related (as it was patient’s opinion as in the previous case 9).

Thoughts

61

The only suspicious looking symptom was the postictal phase lasting up to 10  minutes and also notion from the family about “not being himself” for the rest of the day. Also a non epileptic seizure or stress related incident would probably not result in a road traffic accident (however it can happen). Therefore MRI of the brain was arranged as it was felt that partial/focal seizure with altered awareness could be responsible. From description of the episode it looked that ­consciousness was at least partially preserved (he was able to hear what was happening around).

Figure 10.1  MRI axial T2 weighted image

62

Chapter 10.  Disorientation, Confusion II

Figure 10.2  MRI axial T1 weighted image with Gadolinium contrast

Key Symptoms Likely focal/partial seizure with postictal phase

Answers MRI was obviously pathological with left temporal lobe tumour showing high signal on T2 and low on T1 weighted images (Figs.  10.1 and 10.2). There was no post contrast enhancement (Fig. 10.2). There was some mass effect on the left lateral ventricle (Fig. 10.1, compare with case 9). As there was no enhancement it was most likely a glioma. Patient underwent biopsy and partial resection unfortunately with the conclusion of grade III glioma. Radiotherapy and chemotherapy was initiated as radical removal in this region was impossible (dominant hemisphere etc.). Patient was started

Further Reading

63

on prophylactic antiepileptic treatment (levetiracetam) despite no further seizures were recorded (higher risk of having further seizure with intracranial focal lesion). Keywords  Focal seizures · Temporal lobe · Glioma

Hallmarks Indication of MRI imaging in this particular case can seem questionable but postictal phase and circumstances of patient’s episode justified the request. Interestingly the neurological examination was not particularly helpful and diagnostic thoughts were based on clinical description (which is generally the case in epilepsy). The patient somehow accepted and was actively proposing stress as the cause of his episode, which was misleading. Previous case (case 9) and this case show very similar location of tumour but very different clinical presentation and unfortunately very different prognosis and outcome. Based on literature a median survival of patient with high grade glioma (III) oscillates between 24 and 60 months (for grade IV it is usually less than 24 months).

Further Reading Englot DJ, Chang EF, Vecht CJ. Epilepsy and brain tumors. Handb Clin Neurol. 2016;134:267–85. Kasper DL, Fauci AS, Hauser S, Longo D, Larry Jameson J, Loscalzo J.  Harrisons manual of medicine. 19th ed. New York: McGraw Hill Professional; 2016. Lamborn KR, Chang SM, Prados MD.  Prognostic factors for survival of patients with glioblastoma: recursive partitioning analysis. Neuro-Oncology. 2004;6:227–35. Louis DN, Perry A, Reifenberger G, von Deimling A, Figarella-­ Branger D, Cavenee WK, Ohgaki H, Wiestler OD, Kleihues P, Ellison DW.  The 2016 World Health Organization classification of tumors of the central nervous system: a summary. Acta Neuropathol. 2016;131:803–20. Mumenthaler M, Mattle H. Neurology. Stuttgart: Thieme; 2011. Nadgir R, Yousem DM. Neuroradiology: the requisites. Philadelphia: Elsevier Health Sciences; 2016.

64

Chapter 10.  Disorientation, Confusion II

Notes

Chapter 11 Woman with Sudden Headache

46-year-old lady had a long history of migraines for many years. Headaches were usually linked to menstrual cycle and usually well controlled by sumatriptan, diclofenac and propanolol (as preventative agent it reduced the frequency of headaches). She required diamorphine very rarely. She presented with a headache which she felt was “different” in nature. The previous night she was woken from sleep by the pain at around 3 am (this never occurred with her migraines) and took sumatriptan with minimal benefit or response. She described “feeling certain she was going to die” as her headache was very severe. She came to A&E with conclusion of left sided hemicranial and frontal pain. Objective examination was within normal limits with symmetrical reflexes, normal cranial nerves and no cerebellar signs. Only pathology found later was difference in pupil sizes. The left was smaller than right; sluggish but responding to light. She was hypertensive (156/106). Treating physician referred her for a non contrast CT brain which did not show any obvious pathology (no intracranial bleed). She eventually responded to opiates and her diagnosis was concluded as hemicrania/migraine. In following 48  hours her pain transformed into painful hypoesthesia of the scalp on the left (numb pain reported by patient). Asymmetry of pupils persisted. Patient was reviewed by neurologist 3 days later.

© Springer Nature Switzerland AG 2019 O. Dolezal, Clinical Cases in Neurology, In Clinical Practice, https://doi.org/10.1007/978-3-030-16628-1_11

65

66

Chapter 11.  Woman with Sudden Headache CN, cerebellar normal

+

+

5/5

5/5

+

+

+ Neg. 5/5

+ Neg. 5/5

Thoughts In this case the important question to address is if you would do anything here differently? Was diagnosis of primary headache (it means not linked to any other pathology) correct or could it be headache secondary to another pathology? In your practice you often face patients with ongoing chronic headaches or regular headaches who tell

Thoughts

67

you (with different level of urgency) that “this headache is different”. New type or presentation of headache in patients with known migraine/headache is relatively difficult to tackle. But we should always pay attention to patient’s subjective side of the story. We can make a mistake and tend to explain all headaches (original and new) by original diagnosis. This case is a prime example of a young lady with known migraines and no risk factors. Hemicranial distribution, despite new, is not particularly surprising in migraines or cluster headaches, but other pathologies such as subarachnoid, intraparenchymal bleed or other focal pathology such as infection or sinus thrombosis could also be plausible explanations. Negative CT unfortunately does not mean much in this context. One might argue that asymmetry of pupils is worrying but rationale of our colleagues at A&E that it is related to pain, medication or being habitual was somehow understandable. This asymmetry associated with pain did not fit sidewise (left miotic, right mydriatic, pain on the left side). We know for example that in cilliospinal reflex (pupillary-skin reflex) which tests sympathetic pathways, there is ipsilateral (or bilateral) brief mydriasis (not miosis like here) as representation of sympathetic activation by pain. Thinking more about her pupils as the only objective pathology another option would arise. It could be Horner’s syndrome (opposite mechanism to cilliospinal reflex as sympathetic fibres/axons are damaged). Similar findings (hemicranial headache, and usually neck pain which was not present here) with Horner’s can be seen in carotid dissection. Subarachnoid bleed remained an option as negative CT should be evaluated only in combination with lumbar puncture when we have clinical suspicion. We also know that previous history of migraine can be a risk factor for intracranial bleed (intraparenchymal and subarachnoid as well). After discussion with patient MRI brain and MR angiogram (focusing on carotid arteries and circle of Willis) were arranged; lumbar puncture was planned as a next step.

68

Chapter 11.  Woman with Sudden Headache

Figure 11.1  CT brain axial image (without contrast)

Figure 11.2  MRI axial FLAIR image

 

Hallmarks

69

Key Symptoms Headache changed in nature (new to patient)

Answers After reviewing CT head it was concluded as normal (Fig.  11.1). MRI was performed almost a week later and found left sided frontal atypical bleed (Fig. 11.2) (by typical we mean hypertensive bleed located in region of “perforating arteries” in basal ganglia). MR angiogram looked normal in intracranial region (circle of Willis) and in carotid and ­vertebrobasilar territory as well. There were no signs of sinus thrombosis or arteriovenous malformation. As there was lack of other pathology and patient clinically felt better it seemed we identified the cause of her new left hemicranial headaches. She probably bled into cavernoma or small AVM.  Conservative approach was recommended. Patient’s headache settled completely within 2  weeks and she has remained clinically stable in long-term. Keywords  Headaches · Intraparenchymal bleed · Adie pupil

Hallmarks This case demonstrated three important points. Firstly to rule out bleed (mostly subarachnoid) brain scan (CT) on its own does not seem to be reliable enough (false negative findings in smaller bleeds) and scan always should be evaluated alongside lumbar puncture (if there is convincing clinical ­suspicion). It is true that in this case as there was an intraparenchymal bleed CSF analysis could be normal as well. Frontal location of hematoma is relatively uncommon as mentioned above (approx. 15% of all bleeds). Secondly it is very important to listen to patient’s side of the story and not to be blinded by original diagnosis. Even in known migraneurs we should always consider other causes (bleed, tumour, carotid dissection etc.) especially when there are new (or previously ignored) neurological

70

Chapter 11.  Woman with Sudden Headache

symptoms or headaches have changed in nature. In every patient with chronic headache scanning should be considered at least once in life (preferably MRI), especially when headaches change in nature. Thirdly her pupils were likely complete red herring. She was eventually reviewed by Ophthalmologist (6  months later) with following (very fair) comment: “Neurologist noted uneven pupils at his examination and the right pupil is larger and more or less fixed in mid dilatation with some movements around the pupil sphincter which would be rather consistent with Holmes Adie Syndrome. The left pupil is smaller but it does dilate to some extent when we darken the room. So I don’t think that she has a Horner’s Syndrome but of course I was fortunate enough to know the result of the MRI and angio before I had to give my opinion!” So it seemed that pupil asymmetry was coincidental and could be longstanding.

Further Reading DeMyer W. Technique of the neurologic examination. 5th ed. New York: McGraw Hill Professional; 2003. Kurth T, Kase CS, Schürks M, Tzourio C, Buring JE. Migraine and risk of haemorrhagic stroke in women: prospective cohort study. BMJ. 2010;341:c3659. Lindsay KW, Bone I, Fuller G.  Neurology and neurosurgery illustrated e-book. St. Louis: Elsevier Health Science; 2010. Morgenstern LB, Hemphill JC 3rd, Anderson C, et  al. Guidelines for the management of spontaneous intracerebral hemorrhage: a guideline for healthcare professionals from the American Heart Association/American Stroke Association. Stroke. 2010;41:2108–29.

Notes

Notes

71

Chapter 12 First Seizure

22-year-old man was referred to Neurology outpatient clinic for an isolated epileptic seizure. Seizure occurred in sleep. He was a keen athlete (footballer and rugby player) with no history of any previous illnesses. Seizure occurred in the morning hours on a Sunday. He admitted that the day before he had travelled to capital for an international football game (as a fan, not a player) and since noon had not eaten well and drank some beers (approx. 15  units) between 1  pm till approximately 8  pm. He arrived home around 11  pm and despite feeling hungry he did not eat. He went to bed at 2 am. Around 6  am his girlfriend felt that the “bed was shaking” and she heard a thud as patient fell from his bed. There was whole body jerking which settled after 2–3  minutes. There was convincing postictal phase lasting 15–20  minutes. He remembered paramedics being in his room (so amnesia was not longer than 20  minutes). There was no incontinence or bitten tongue. He was found to have low sugar (2.1 mmol/l). When questioned he did not mention any collateral family or personal medical history suggestive of epilepsy or genetic problems (no illnesses “running in the family”). His girlfriend mentioned that for last 6 months he had had intermittent left hand jerks he can not control but they were of no concern to him. These jerks were always on one side. Neurological exam was completely normal, with symmetrical reflexes and normal cranial nerves, EOMs were also normal. There was no sensory deficit and cerebellar functions were normal. © Springer Nature Switzerland AG 2019 O. Dolezal, Clinical Cases in Neurology, In Clinical Practice, https://doi.org/10.1007/978-3-030-16628-1_12

73

74

Chapter 12.  First Seizure CN, cerebellar normal

+

+

5/5

5/5

+

+

+ Neg. 5/5

+ Neg. 5/5

Thoughts This case is rather typical- young individual with first epileptic seizure after the night out. This is very common scenario seen in general neurology clinic and GP practice. Typically we see provoked seizures (alcohol withdrawal, sleep deprivation) occurring on Saturdays or Sundays respectively as most

Thoughts

75

“partying” usually occurs on Fridays and Saturdays. There is always the conundrum how far we should take our effort regarding further tests. General rule is that if there is unprovoked seizure and persistent focal neurology we should consider seizure being secondary to other brain pathology. If above description was three lines shorter then I would agree that no further action was needed; young healthy man with significant triggers factors (sleep deprivation, exhaustion, alcohol, hypoglycaemia) having isolated seizure. However the one symptom his girlfriend mentioned is rather atypical. It is true that myoclonic jerks raise the possibility of juvenile myoclonic epilepsy (JME) but presence only on one side of the body raises the question about focal process. It was also obvious that this was his first epileptic seizure (JME usually starts earlier in life – between the age of 5–16). From description of the seizure (semiology) it was impossible to judge if seizures were primary generalised or secondary generalised with focal onset, therefore MRI of brain and EEG were requested.

Figure 12.1  MRI sagittal T1 weighted image with gadolinium contrast

76

Chapter 12.  First Seizure

Figure 12.2  MRI sagittal T1-FLAIR image

Figure 12.3  MRI axial FLAIR image

 

Hallmarks

77

Key Symptoms Focal myoclonus, Epileptic seizure

Answers MRI imaging revealed unexpected pathology – a likely low grade glioma was found in the posterior part of the medulla and extending into the foramen of Magendie, measuring approximately 22 mm × 20 mm × 20 mm (Figs. 12.1, 12.2, and 12.3). There was no hydrocephalus. On the ­T2-weighted/FLAIR scan this lesion is of high signal intensity (Fig. 12.3) and it did not enhance after contrast (Fig. 12.1). Surgery/biopsy at that location was not possible. Chemo and radiotherapy was discussed with patient. Clinically this finding was unexpected and likely not related to patient’s seizure and myoclonus and was purely coincidental. We can claim that his myoclonus could originate from medulla but this is purely an assumption. Neurosurgeon commented that “this patient is a case of VOMIT”. This is questionable as VOMIT means Victim of Medical/or Modern Imaging Technology (despite it led to a diagnosis). Understandably the diagnosis had very detrimental psychological and emotional effect on the wellbeing of this young patient. As our therapeutic options are limited his prognosis is uncertain. Keywords  Focal myoclonus · Isolated seizure · Provoked seizure · Glioma

Hallmarks This case clearly demonstrated that MRI imaging/brain scan can lead to unexpected results and it is essential to provide radiologist with honest clinical information (where to look for the lesion, what is assumed pathological process, relevant co-morbidities etc.) and to be clear what is expected from the scan. MRI scan in this case was not indicated wrongly (rather overcautiously) as MRI request was backed by clinical findings (left arm myoclonus).

78

Chapter 12.  First Seizure

Further Reading Claus EB, Walsh KM, Wiencke JK, Molinaro AM, Wiemels JL, Schildkraut JM, Bondy ML, Berger M, Jenkins R, Wrensch M.  Survival and low-grade glioma: the emergence of genetic information. Neurosurg Focus. 2015;38:E6. Englot DJ, Chang EF, Vecht CJ. Epilepsy and brain tumors. Handb Clin Neurol. 2016;134:267–85. Gupta SN, Belay B.  Intracranial incidental findings on brain MR images in a pediatric neurology practice: a retrospective study. J Neurol Sci. 2008;264:34–7. Håberg AK, Hammer TA, Kvistad KA, Rydland J, Müller TB, Eikenes L, Gårseth M, Stovner LJ.  Incidental intracranial findings and their clinical impact; the HUNT MRI study in a general population of 1006 participants between 50-66 years. PLoS One. 2016;11:e0151080. Hayward R.  VOMIT (victims of modern imaging technology)–an acronym for our times. BMJ. 2003;326:1273. Louis DN, Perry A, Reifenberger G, von Deimling A, Figarella-­ Branger D, Cavenee WK, Ohgaki H, Wiestler OD, Kleihues P, Ellison DW.  The 2016 World Health Organization classification of tumors of the central nervous system: a summary. Acta Neuropathol. 2016;131:803–20. Nadgir R, Yousem DM. Neuroradiology: the requisites. Philadelphia: Elsevier Health Sciences; 2016. Schomas DA, Issa Laack NN, Rao RD, Meyer FB, Shaw EG, O’Neill BP, Giannini C, Brown PD.  Intracranial low-grade gliomas in adults: 30-year experience with long-term follow-up at Mayo Clinic. Neuro-Oncology. 2009;11:437–45.

Notes

Notes

79

Chapter 13 Elephant in the Room vs. Red Herring

67-year-old gentleman was referred to Neurology outpatient department for a 6 month history of gait difficulties, incontinence and occasional swallowing issues. He also reported a numb right side of his face (“like having local anaesthesia at the dentist”) and that his symptoms were gradually progressing. He was referred by his Haematologist as he had a history of Chronic Lymphocytic leukaemia (CLL) with lymphadenopathy, splenomegalia and leukocytosis. This diagnosis was known for more than 5  years (verified by bone marrow biopsy) and clinically he seemed stable from a haematology point of view. On objective examination there was gait disturbance (rather spastic gait) with bilateral legs weakness and increased reflexes/ hyperreflexia and positive plantar responses (spastic upper motor neurone paraparesis). There were no obvious fasciculations or muscle wasting. There were also sphincter issues  – retention with overflow incontinence. Findings in upper extremities were normal. Cranial nerves were normal. There was only right sided facial hypoesthesia (light touch, pinprick) in all three divisions of trigeminal nerve (white area on the diagram). Soft palate and tongue were moving sufficiently. EOMs were normal. Other sensory functions were within normal limits (only slightly reduced vibration sense in ankles).

© Springer Nature Switzerland AG 2019 O. Dolezal, Clinical Cases in Neurology, In Clinical Practice, https://doi.org/10.1007/978-3-030-16628-1_13

81

82

Chapter 13.  Elephant in the Room vs. Red Herring Rest of CN, cerebellar functions normal,

+ +

+ +

5/5

5/5

+ +

+ +

+ + pos 4+/5

+ + pos 4/5

Thoughts This case represents a patient with multiple neurological signs. Paraparesis/leg weakness and bladder issues are suggestive of spinal cord involvement; likely below cervical intumescence as arms and hands’ functions were not impaired (only increased reflexes but no paresis/weakness, sensory deficit or muscle wasting etc.). Right sided facial numbness/hypoaesthesia is suggestive of trigeminal (V) nerve or brainstem (pons) involvement. Brainstem lesion, if extensive enough, can be responsible for paraparesis as well but one would expect more obvious signs in upper extremities and more of cranial nerves

Thoughts

83

to be affected (III, IV, VI, VII, IX, X and XII seemed normal as EOMs, facial muscles, soft plate and tongue functions were unaffected). At the point of initial examination it was not certain how to interpret his occasional swallowing issues. It was also possible that lesions could be multilevel. Obviously his symptoms could be associated with longstanding haematological diagnosis (“elephant in the room”) or could be purely coincidental (so it would make diagnosis of leukaemia a “red herring”). In haematological illnesses you should be always aware of possible neurological complications. The most common type of neurological ­symptoms in haematological malignancies are represented by a) local infiltration (meninges, brain parenchyma, nerve roots etc.) but also b) paraneoplastic symptoms (polyneuropathy, encephalopathy of autoimmune nature (anti-Hu, Yo, Ri Abs positivity, NMDA receptor, voltage gated calcium and potassium channel antibodies (anti VGCC, VGKC Abs) etc.). Paraneoplastic syndromes are very difficult to diagnose as they can pre-date definite cancer diagnosis by months or years (e.g. Lambert Eaton myasthenic syndrome, as the most legendary example of paraneoplastic syndrome). We must not forget that also degenerative disorders such as anterior horncell degeneration/motor neurone disease (e.g. amyotrophic lateral sclerosis (ALS)) can be a diagnostic possibility (however sensory deficit would be very unlikely). Very interesting option here was progressive multifocal encephalopathy (PML) caused by JC virus which is known to be more frequent in oncology, haematology and generally in immunosupresed patient (HIV/AIDS). Interestingly in this particular case there was diagnostic argument from the beginning between specialties  - Neurologist thought that there is a connection between current symptoms and CLL and Haematologist wanted to rule out other, rather primary neurological, causes. After mutual agreement, MRI brain with contrast and whole spine (as there was clinical suspicion that we are dealing with multilevel multiple lesions) was arranged. LP was intended to be done after the scan. JC virus test was also requested (serology from blood and PCR from CSF). If above tests were not conclusive, nerve conduction studies/electromyography was planned to rule out neurodegenerative disorder.

84

Chapter 13.  Elephant in the Room vs. Red Herring

Figure 13.1  MRI sagittal FLAIR image

Figure 13.2  MRI sagittal T2 weighted image

Thoughts

85

Figure 13.3  MRI axial T1 weighted image with gadolinium contrast

Figure 13.4  MRI sagittal T1 weighted image with gadolinium contrast

86

Chapter 13.  Elephant in the Room vs. Red Herring

Figure 13.5  MRI sagittal T2 weighted image

Key Symptoms Paraparesis, retention with overflow incontinence, facial hypoasthesia

Answers Imaging was strikingly pathological and the assumption, based on neurological examination, that there would be multiple lesions was proven correct. First two scans (Figs. 13.1 and 13.2) show midbrain, pons and medulla being involved. Other two scans (Figs. 13.3 and 13.4) clearly demonstrated gadolinium enhancement of lesions. Similar lesion was also found in epiconus and conus medullaris (Fig. 13.5). MRI findings completely explained patient’s symptoms so other aetiology and differential diagnoses such as MND/ ALS were dismissed. Unfortunately these results did not lead to diagnostic consensus. Conundrum between primary



Second Thoughts

87

neurological or primary haematological pathology persisted. Patient looked very stable from haematological point of view. To complicate things further, patient expressed his opinion that other diagnoses such as demyelination could be occurring. CNS infiltration is a very rare complication of CLL ­(haematology argument) but reported and known. In this particular case patient’s age would be very atypical for demyelination. Radiologists were leaning towards CLL. Lumbar puncture was performed and found high protein 1.2  g/l, eight white cells (lymphocytes) and negative oligoclonal bands (OCBs). Qualitative cytology was inconclusive (malignant cell would be likely CD5+ B cell which looks very similar to normal lymphocyte (see Harrison’s manual of medicine in further reading section). PML was very unlikely as PCR (from CSF) and serology from blood were negative. Second round of discussion occurred again. Patient’s condition seemed stable. High dose steroids (methylprednisolone) were agreed on in the meantime with significant clinical response (improvement in mobility). CT chest and abdomen found stable known lymphadenopathy and splenomegaly. Keywords  Chronic lymphocytic leukaemia · CNS infiltration · Demyelination · Oligoclonal bands · Myelopathy

Second Thoughts Lumbar puncture results went once more against demyelination/multiple sclerosis and were rather in agreement with CLL as the culprit. Response to steroids is typical for both conditions so it was not particularly helpful marker. Negative OCBs also go against demyelination (but it is true OCBs can be falsely negative at times). Biopsy was arranged as it was agreed at Multidisciplinary Team (MDT) meeting. Histology revealed CLL infiltration with diagnostic certainty. Chemotherapy was started with clinical improvement.

88

Chapter 13.  Elephant in the Room vs. Red Herring

Hallmarks This case demonstrates rare complication of relatively common haematological disorder (especially in older individuals), but there are some basic points to pay attention to. The most important point here was that the clinical picture and objective examination reliably suggested areas which may be affected, which was proven by MRI.  Multidisciplinary approach in these cases is essential as the diagnostic consensus reached. Multiple sclerosis is extremely rare in patients over 60 years of age. Stability of any oncological disease on imaging and laboratory tests provides only limited reassurance when suspecting central nervous system spread and even less so in paraneoplastic syndromes.

Further Reading Benjamini O, Jain P, Schlette E, Sciffman JS, Estrov Z, Keating M.  Chronic lymphocytic leukemia with central nervous system involvement: a high-risk disease? Clin Lymphoma Myeloma Leuk. 2013;13:338–41. Kasper DL, Fauci AS, Hauser S, Longo D, Larry Jameson J, Loscalzo J.  Harrisons manual of medicine. 19th ed. New York: McGraw Hill Professional; 2016. Lindsay KW, Bone I, Fuller G.  Neurology and neurosurgery illustrated e-book. St. Louis: Elsevier Health Sciences; 2010. Nadgir R, Yousem DM. Neuroradiology: the requisites. Philadelphia: Elsevier Health Sciences; 2016. Sanchez-Quintana A, Breña-Atienza J, Marrero-Santos C, Alvarez-­ Acosta L. Late relapse of progressive multifocal leucoencephalopathy postallogenic transplant in a young patient with CLL. BMJ Case Rep. 2013; https://doi.org/10.1136/bcr-2013-200213. Thompson AJ, Banwell BL, Barkhof F, et al. Diagnosis of multiple sclerosis: 2017 revisions of the McDonald criteria. Lancet Neurol. 2018;17:162–73.

Notes

Notes

89

Chapter 14 Gait Disorder and Falls

66-year-old gentleman was admitted to the hospital following a fall. He was a gentleman with a history of atrial fibrillation (AF) on warfarin, hypertension and diabetes. So far AF has responded well to “rate control” treatment (previously on beta blocker, currently on calcium channels blocker). He complained about intermittent dizzy spells and was seen by Cardiology. However the history surrounding the fall (and how he had actually arrived to hospital) was a bit vague. On the day of admission he was walking to his GP surgery to have his INR checked as it had been raised the previous day (4.2). He suddenly felt very dizzy and fell over. He did not lose consciousness. He eventually managed to get to surgery and had an ECG which showed AF with fast heart rate of about 150 bpm. It transpired that he had been unsteady on his feet “for months” and used a stick for walking now (six months ago he was walking unsupported). There was also urinary urgency with intermittent urinary incontinence. His wife who arrived later was not aware of any other issues; however she admitted that he was generally “slower”. On a previous tilt test, cardiologist found postural/orthostatic hypotension. His case on admission was concluded (working diagnosis) as unexplained gait disorder, falls with orthostatic features (likely cardiogennic), hypertension, type II diabetes, diabetic retinopathy, diabetic foot, and atrial fibrillation. He was not examined by neurologist. After his tachycardia improved on changed medication, he wished to be discharged without patient follow-up. © Springer Nature Switzerland AG 2019 O. Dolezal, Clinical Cases in Neurology, In Clinical Practice, https://doi.org/10.1007/978-3-030-16628-1_14

91

92

Chapter 14.  Gait Disorder and Falls

Thoughts Approaching similar patient in A&E setting can be difficult. This patient had known cardiac/vascular issues and his fall seemed to represent syncope or rather pre-syncope. There were no seizure markers (no bruising, obvious injury, no loss of consciousness, no postictal confusion as patient was able to finish his journey to GP practice etc.). We can argue that diabetes can lead to autonomic dysfunction (neuropathy) so it would make orthostatic collapses even more expected. Under circumstances diagnostic conclusions made sense.

Second Visit Patient was referred to the Neurology outpatient clinic 2 months later for headaches and prominent memory problems. Previous admission was discussed in detail. He had not had any further major dizzy spells but was complaining about mild ongoing dizziness. It seemed that his walking problems were slowly progressing and the cognitive “slowness” mentioned 2  months ago had progressed too. During examination paraparesis (moderate weakness) with increased reflexes and positive plantars (so upper motor neurone in nature) was found. Mini mental state examination (MMSE) was 23/30. There was urinary urgency including nocturia and ­incontinence (more prominent than before). Cranial nerves and cerebellar functions were normal. His gait was broad based, slow with shortened low steps. He walked “cautiously” with one stick but without obvious shuffling. He assumed mildly flexed posture when standing. Muscle tone was slightly increased in legs but there were no signs of rigidity in legs or hands.



Second Thoughts?

93

CN, cerebellar normal

+

+

5/5

5/5

+ +

+ +

+ + Pos. 4+/5

+ + Pos. 4+/5

Second Thoughts? His heart problems can not explain his ongoing walking issues, cognitive decline and bladder problems. Interpreting gait can be difficult as there are many clinical phenotypes  – frontal apraxia of gait, gait of aged, dystonic gait, spastic gait, cerebellar gait/ataxic gait, Parkinsonian gait etc. (please see further reading). But in this particular case we should judge the clinical picture in combination with other symptoms. There is not only abnormal gait but also incontinence and cognitive decline (relatively mild). This triad is well recognised in hydrocephalus. Sometimes headache can be present as well as in this case. Truth

94

Chapter 14.  Gait Disorder and Falls

is that other processes leading to increased intracranial pressure (neoplasm/space occupying lesion) can have similar presentation. You also must not forget that subdural chronic hematoma should be always considered in anticoagulated patients (as seen in case 3). Chronic vascular changes/atherosclerosis could also be a culprit here as there is prominent vascular/cardiac history. Cauda equina syndrome is not suitable explanation here as findings were upper motor neurone (not talking about cognitive deficit), but cervical myelopathy should be also considered. Brain imaging was decided to be the most meaningful way forward. MRI scan of brain and cervical spine (myelopathy) was arranged.

Figure 14.1  MRI axial T1 weighted image with gadolinium contrast

 

Answers

95

Figure 14.2  MRI axial T2 weighted image

Key Symptoms Abnormal gait, incontinence and cognitive decline

Answers An MRI scan found third ventricle colloid cyst (Fig. 14.1), causing obstructive hydrocephalus (especially increased was volume of right lateral ventricle but occipital horn of left ventricle was enlarged as well as seen on Fig. 14.2). Cyst was located at foramen Monro. There was transependymal fluid migration (Fig. 14.2). Case was discussed with neurosurgeons. Endoscopic procedure was performed and patient improved, however incompletely. Last neurology letter a year later reads “I am pleased to report patient has continued to make good clinical progress. He is mobile still with one stick but feels well in himself. He is out and about a fair amount of time. His memory is

96

Chapter 14.  Gait Disorder and Falls

subjectively not great, but he is managing to use a smart phone, and in fact has showed his wife how to use the smart phone…” Keywords  Gait disorder · Cognitive decline · Incontinence · Hydrocephalus · Colloid cyst

Hallmarks Recognising lead symptoms (in this case gait disorder, dizziness, sphincter symptoms, cognitive decline) and not being scotomised by previous diagnosis (cardiac, vascular) is important. Many neurological diagnoses (dementia, degenerative disorders etc.) are not curable or even treatable and that is why conditions such as subdural hematoma or hydrocephalus should be recognised early and treated early (see case 3). There could be as in this case visible improvement but surgery also has progression preventing role.

Further Reading Armao D, Castillo M, Chen H, Kwock L. Colloid cyst of the third ventricle: imaging-pathologic correlation. AJNR Am J Neuroradiol. 2000;21:1470–7. DeMyer W. Technique of the neurologic examination. 5th ed. New York: McGraw Hill Professional; 2003. Jones JHR Jr, Srinivasan J, Allam GJ, Baker RA. Netter’s neurology. Philadelphia: Elsevier Health Sciences; 2011. Nadgir R, Yousem DM. Neuroradiology: the requisites. Philadelphia: Elsevier Health Sciences; 2016. Ropper A, Samuels M, Klein J.  Adams and Victor’s principles of neurology. 10th ed. New York: McGraw Hill Professional; 2014.

 

Notes

Notes

97

Chapter 15 “Tongue Tied” Man with Opiate Addiction

44-year-old man with history of opiate addiction (last 12 month on methadone 40–70 mg/daily) and alcohol problems in the past (admitting up to 15 units a day) was admitted via A&E. He was also treated for depression in the past. At A&E he complained about speech difficulties and right sided headache. He reported his symptoms lasting for last 2 weeks. During that period he drank alcohol extensively so he admitted not to be certain about symptoms duration. He could not rule out neck or head injury as there was significant amnesia linked to alcohol. For the same reason he also could not rule out any loss of consciousness during that period. He was not certain about use of other illicit drugs (e.g. cocaine, heroin, amphetamine). On admission there was obvious dysarthria/slurred speech and dysphagia. There were no obvious traumatic changes on his neck or head. On objective examination bilateral complete hypoglossal nerve palsy was found. Other cranial nerves were normal. There were no other signs of motor or sensory deficit. Pupils were symmetrical and responsive with no signs of Horner’s syndrome. Masseter jerk was absent. Patient was significantly hypertensive on admission (systolic pressure 143–192, diastolic 103– 128.). CRP was 40 and a white blood cell count (WBC) was 14, ALT was minimally increased to 64 iu/l (limit 50) and Gamma GT was 187 iu/l (limit 50), ESR was 22 (limit 15). CT head was normal. Lumbar puncture/CSF analysis showed raised white blood cell count (eight lymphocytes) and © Springer Nature Switzerland AG 2019 O. Dolezal, Clinical Cases in Neurology, In Clinical Practice, https://doi.org/10.1007/978-3-030-16628-1_15

99

100

Chapter 15.  “Tongue Tied” Man with Opiate Addiction

protein (0.53 g/l, upper limit 0.4). Treatment with Acyclovir and Cefotaxim was started. PCR (viral) from cerebrospinal fluid was negative. Methadone metabolites were positive in urine. Speech and language team reviewed patient and swallowing was assessed. Nasogastric tube was not needed. XIIth bilateral palsy

+

+

5/5

5/5

+

+

+ Neg. 5/5

+ Neg. 5/5

Thoughts In this case we have a young patient with obvious risk factors (addiction, alcohol) and with striking neurological and medical signs (hypoglossal bilateral palsy, new onset of hypertension). As you can see from blood tests requested, we considered

Thoughts

101

diagnoses such as neuroinfection (LP done), space occupying lesion/neoplasm (CT performed). We also had to consider toxic aetiology (history of drug abuse). You should think about a possibility of bulbar syndrome of other origin such as degenerative process (bulbar form of motor neurone disease (MND)) or myasthenia gravis (MG) (? b ­ ulbar syndrome). However timing and relatively acute/subacute onset is not suggestive of this. Paraneoplastic origin seems less likely but vasculitic and vascular origin (sudden onset) were possible. Therefore some further blood tests were requested (such as acetylcholine receptor, anti MUSK (MG), ANA, ENA, ANCA, IgG, IgM cardiolipin antibodies (vasculitis screen), anti-neuronal, NMDA receptor, VGKC antibodies (paraneoplastic origin, limbic encephalitis) etc.). All of the above were normal. To be more certain about infratentorial/brainstem involvement (XII nerve) MRI with contrast was requested.

Figure 15.1  MRI axial T2 weighted image

102

Chapter 15.  “Tongue Tied” Man with Opiate Addiction

Figure 15.2  MRI axial T2 weighted image

Figure 15.3  CT angiogram, axial scan

Answers

103

Figure 15.4  CT angiogram coronal scan

Key Symptoms Bilateral hypoglossal palsy

Answers This time MRI provided essential information. First two pictures (Figs. 15.1 and 15.2) represent MRI T2 weighted images showing bilateral “crescent sign” (hyperintense/bright rim suggestive of late subacute haematoma) around carotid artery flow voids (hypointense/dark). Second set of two pictures (Figs.  15.3 and 15.4) shows CT angiogram. Axial scan shows thickening of carotid artery wall more on the left (Fig. 15.3) – much thinner diameter of artery lumen- and tiny dark endothelial “flap” on the right (Fig. 15.3). False lumen is seen on the right on the last scan (Fig. 15.4). These findings led to a diagnosis of bilateral carotid artery dissection. When we look into anatomy textbooks we can see that relationship

104

Chapter 15.  “Tongue Tied” Man with Opiate Addiction

between carotid artery and hypoglossal nerve in the neck is variable but it is close. It is somehow unexpected because intracranially carotid artery and hypoglossal nerve appear very much apart (posterior fossa and brainstem blood supply is covered by mainly vertebral/ basilar arteries). Hypoglossal nerve crosses carotid artery above carotid bifurcation (so crosses both internal and external carotid artery). Therefore symptoms were caused by bilateral internal carotid artery dissection. Dissection was probably outcome of head/neck trauma (compression occurring when lying on the hard floor unconscious, probably not in the most physiological position) and decompensated hypertension. Horner’s syndrome, headache and neck pain would be typical carotid dissection symptoms but were clearly absent in this case (except headache). In this case hypoglossal nerve palsy improved dramatically (persistent left sided hypoglossal palsy (moderate) and normal right hypoglossal nerve functions was seen after 3 months since hospital discharge). His blood pressure remained stable on ACE inhibitor and calcium channel blocker. Keywords  Bilateral hypoglossal palsy · Carotid dissection

Hallmarks Bilateral hypoglossal palsy of this origin is extremely rare and even unilateral hypoglossal nerve palsy is uncommon. In hypoglossal nerve palsy you should suspect brainstem and meningeal process (vascular, inflammatory, neoplastic), infection (e.g. TB meningitis), neuromuscular junction’s problem (MG) and degenerative disorder (MND). This case showed very rare cause of hypoglossal nerve palsy but clearly demonstrated that good knowledge of anatomical landscape in combination with neurological examination can lead to reliable identification of the level of the lesion. Another clinical point is that hypoglossal nerves can be injured during carotid endarterectomy as well.

Notes

105

Further Reading DeMyer W. Technique of the neurologic examination. 5th ed. New York: McGraw Hill Professional; 2003. Epstein E, Khan MA, Francis D, Sada P, Thuse M.  Carotid artery dissection causing hypoglossal nerve palsy. BMJ Case Rep. 2012; https://doi.org/10.1136/bcr.01.2012.5636. Kim T, Chung S, Lanzino G.  Carotid artery-hypoglossal nerve relationships in the neck: an anatomical work. Neurol Res. 2009;31:895–9. Nadgir R, Yousem DM. Neuroradiology: the requisites. Philadelphia: Elsevier Health Sciences; 2016.

Notes

Chapter 16 Right Sided Hemiparesis in Young Smoker

40-year-old man was admitted for right sided weakness of his arm and leg. He walked with a limp. Following discussion with the patient and his family, onset of symptoms seemed subacute and progressive. Symptoms started “weeks ago” but he was “getting worse”. The patient had smoked 30–40 cigarettes every day for more than two decades. He denied any head or neck injury. On objective examination there was hyperreflexia on the right with positive plantars bilaterally. His right sided weakness (hemiparesis) was relatively severe (3/5) and it seemed to be upper motor neurone in origin. His right calf (m. gastrocnemius) appeared wasted and atrophic. There were no sensory or bladder symptoms. Cranial nerves (including facial nerves) and cerebellar functions were normal.

© Springer Nature Switzerland AG 2019 O. Dolezal, Clinical Cases in Neurology, In Clinical Practice, https://doi.org/10.1007/978-3-030-16628-1_16

107

108

Chapter 16.  Right Sided Hemiparesis in Young Smoker CN, cerebellar normal

+ +

+

3/5

5/5

+ +

+

+ + Pos. 3/5

+ Pos. 5/5

Thoughts Hemiparesis as a symptom is always suspicious of being caused by a hemispherical lesion, most commonly stroke (likely ischemic). There seemed to be an obvious risk factor (smoking). Slightly confusing in this story was the progressive nature of symptoms, facial nerve sparing and also the patient’s age. Regardless of this, vascular aetiology had to be considered and a CT scan was requested and was normal.

Thoughts

109

After weeks of symptoms you would expect ischemia to be clearly visible especially when we consider the relative severity of his weakness. Anatomically it seemed that the problem was in corticospinal tracts or contralateral (left) motor area. Age, objective findings and progressive nature could be suggestive of demyelination (primary progressive multiple sclerosis) in the same area (or ipsilateral cervical spinal cord involvement). Muscle atrophy would be very surprising and clinically did not fit with upper motor neurone process. It appeared that there was a mixture of lower and upper motor neurone involvement. This could be suggestive of neurodegenerative disorder (Motor Neurone Disease (MND)- amyotrophic lateral sclerosis (ALS), primary lateral sclerosis (PLS) etc.). To investigate these possibilities an MRI brain, cervical and thoracic spine was arranged (stroke, demyelination, myelopathy) and LP was performed. Nerve conduction studies were requested as well.

Figure 16.1  MRI coronal FLAIR image

110

Chapter 16.  Right Sided Hemiparesis in Young Smoker

Figure 16.2  MRI sagittal FLAIR image

Key Symptoms Hemiparesis, gastrocnemius atrophy

Answers MRI of the brain was not suggestive of demyelination and spinal cord was normal. There was an interesting finding of high signal intensity on T2 and FLAIR sequences within caudal third of the posterior limb of the internal capsule (Figs. 16.1 and 16.2), extending down to the level of the cerebellar peduncle. This could represent normal variation; however corticospinal tract degeneration such as in ALS can have similar appearance. When this finding was put into clinical context, this case suddenly made more sense. In motor neurone disease (e.g. ALS) we usually see mixture of upper and lower motor neurone findings (hyperreflexia and muscle wasting). Nerve conduction studies

Further Reading

111

found widespread denervations even on clinically unaffected left side. Lumbar puncture was unremarkable. Sadly patient gradually progressed and died within 2 years of diagnosis. Keywords  Hemiparesis · Motor neurone disease · ALS

Hallmarks In this particular case the neurological examination played the most important role. Mixed signs were suggestive of motor neurone disease. Electrophysiology concluded this diagnosis. Another learning point here is that not every hemiparesis is a stroke and other explanations should be considered especially in younger adults. But it would be fair to say that MND in young adults is extremely rare and primary progressive demyelination (multiple sclerosis) and even the stroke (when we take into account patient’s risk factors) would be statistically more likely.

Further Reading Ferraro PM, Agosta F, Riva N, Copetti M, Spinelli EG, Falzone Y, Sorarù G, Comi G, Chiò A, Filippi M. Multimodal structural MRI in the diagnosis of motor neuron diseases. Neuroimage Clin. 2017;16:240–7. Foster LA, Salajegheh MK.  Motor neuron disease: pathophysiology, diagnosis, and management. Am J Med. 2018; https://doi. org/10.1016/j.amjmed.2018.07.012. Jin J, Hu F, Zhang Q, Jia R, Dang J. Hyperintensity of the corticospinal tract on FLAIR: a simple and sensitive objective upper motor neuron degeneration marker in clinically verified amyotrophic lateral sclerosis. J Neurol Sci. 2016;367:177–83. Nadgir R, Yousem DM. Neuroradiology: the requisites. Philadelphia: Elsevier Health Sciences; 2016. Ropper A, Samuels M, Klein J.  Adams and Victor’s principles of neurology. 10th ed. New York: McGraw Hill Professional; 2014.

112

Chapter 16.  Right Sided Hemiparesis in Young Smoker

Notes

Chapter 17 Young Man with Pins and Needles

20-year-old man was referred to neurology clinic for pins and needles in both hands. Symptoms started 6  months ago. Symptoms were provoked by physical activity and became much worse after busy day; he worked hard as a joiner. He was referred as a suspected diagnosis of multiple sclerosis (MS). He did not have any other symptoms. On objective examination reflexes were present and not increased; apart from absent ankle jerks bilaterally. Plantars were negative. Muscle strength was normal. There were no sphincter symptoms. He did not have any symptoms on examination but paresthesias in his hands did not seem to respect any particular dermatome. Vibration sense was reduced in legs (ankle level) but light touch and pin prick were normal. Cranial nerves were normal and there was no ataxia.

© Springer Nature Switzerland AG 2019 O. Dolezal, Clinical Cases in Neurology, In Clinical Practice, https://doi.org/10.1007/978-3-030-16628-1_17

113

114

Chapter 17.  Young Man with Pins and Needles CN, cerebellar normal

+

+

5/5

5/5

+

+

– Neg. 5/5

– Neg. 5/5

Thoughts The usual scenario is that almost all symptoms in young adults are suspected to be due to demyelination (compare with case 21). MS would be definitely on top of the list of dif-

Thoughts

115

ferentials, but do objective findings represent central/upper motor neurone pathology? Short answer would be: “No”. Reduced reflexes (even areflexia) in somebody of this age should lead us rather to a peripheral origin (lower motor neuron, peripheral nerve). The same is valid for intermittent nature, short term duration and absence of other symptoms (e.g. sphincters). Proposing polyneuropathy in somebody of this age would not seem reasonable or expected, not only because of age but onset in upper extremities is unusual for polyneuropathy. MRI brain and C spine were requested as well as blood tests including neuropathy screen (vasculitis screen, serum protein electrophoresis, Folate, B12 etc.). Lumbar puncture was also planned.

Figure 17.1  MRI sagittal T2 weighted image

116

Chapter 17.  Young Man with Pins and Needles

Figure 17.2  MRI sagittal T2 weighted image

Key Symptoms Areflexia, Paresthesias/sensory symptoms

Answers Blood analysis revealed a low B12 of 82 pmol/l (range 110– 664 pmol/l) which would lead us to polyneuropathy as a diagnosis. MRI brain was completely normal with no signs of inflammation/demyelination but MRI of the cervical spine showed a long lesion between C1 and C6 segments (Fig. 17.1). This lesion dominantly involved the dorsal columns. Findings were discussed with a radiologist with the conclusion of combined degeneration of the spinal cord, likely on the basis of B12 deficiency. Differential diagnosis of demyelination was still considered. Neuromyelitis optica (NMO)/aquaporine4

Further Reading

117

antibodies were negative. These findings were discussed with the patient and his family and it transpired that his paternal grandfather was followed (and treated successfully) for B12 deficiency polyneuropathy by our department for years. Intrinsic factor antibodies were negative in this patient and his granddad as well. Patient was not keen to proceed to LP. He was started on B 12 substitution and clinically started to improve and 3  weeks later he felt asymptomatic. MRI 6  months later showed complete resolution of findings (see below Fig. 17.2). Keywords  B12 deficiency · Spinal cord · Mixed degeneration · Dorsal columns

Hallmarks This case again demonstrates that clinical picture is essential for test interpretation. MRI imaging on its own was only partially helpful and had to be contextualised. It is important to know that not every hyperintensity on MRI indicates demyelination. Family history also provided important piece of information.

Further Reading Karantanas AH, Markonis A, Bisbiyiannis G.  Subacute combined degeneration of the spinal cord with involvement of the anterior columns: a new MRI finding. Neuroradiology. 2000;42:115–7. Nadgir R, Yousem DM. Neuroradiology: the requisites. Philadelphia: Elsevier Health Sciences; 2016. Ropper A, Samuels M, Klein J.  Adams and Victor’s principles of neurology. 10th ed. New York: McGraw Hill Professional; 2014. Yamada K, Shrier DA, Tanaka H, Numaguchi Y.  A case of subacute combined degeneration: MRI findings. Neuroradiology. 1998;40:398–400.

118

Chapter 17.  Young Man with Pins and Needles

Notes

Chapter 18 Painful and Weak Young Man

27-year-old man was admitted with a severe occipital headache, acute in onset. He also complained of very severe bilateral leg and arm pain. Reflexes were increased and plantars negative. Cranial nerves and cerebellar functions were normal. Sphincters were normal. Rest of neurological examination was influenced by serious pain in all four extremities (showed on diagram), therefore patient was very difficult to examine. He had a dry cough. CRP was marginally increased (25). Hi temperature was increased (37.4 °C). During admission his pain became more prominent in arms and forearms, suggestive of radiculopathic pain.

© Springer Nature Switzerland AG 2019 O. Dolezal, Clinical Cases in Neurology, In Clinical Practice, https://doi.org/10.1007/978-3-030-16628-1_18

119

120

Chapter 18.  Painful and Weak Young Man CN, cerebellar normal

+ +

+ +

5/5

5/5

+ +

+ +

+ + Neg. 5/5

+ + Neg. 5/5

Thoughts In this case we are dealing with a generally unwell young man. In a similar case (acute onset of symptoms, headache, pyrexia etc.) infection should be always considered. A CT head was arranged, with a normal result. Lumbar puncture is essential in similar cases therefore was arranged. Pain became more prominent in upper arms and clinically it looked like radicular pain (mainly affecting C6, C7, C8 bilateral but initially in lumbosacral region as well). Infectious viral meningitis with radiculitis or radiculomyelitis (involving not only roots but spinal cord as well) could be a possible diagnosis. Chest X-ray was requested and reported clear. To rule out other inflammatory processes neck/cervical spine and brachial plexus MRI were requested. NCS were arranged as well.

Thoughts

121

Figure 18.1  MRI sagittal T1 weighted image with gadolinium contrast

Figure 18.2  MRI sagittal T1 weighted image with gadolinium contrast

122

Chapter 18.  Painful and Weak Young Man

Key Symptoms Neuropathic radicular pain, headaches

Answers CSF analysis showed increased protein of 1.2 g/l and 56 WBC (lymphocytes and monocytes). Patient was started on antivirals (acyclovir and later on valacyclovir). At this point he was treated as viral meningitis with probable radiculitis. His cough and respiratory functions remained stable over the following 24 hours and his temperature settled. PCR from CSF later showed herpes simplex virus (HSV 2) positivity. However despite antiviral treatment, his pain did not settle. Radiculomyelitis was suspected as a neurological manifestation of HSV-2 infection. On MRI there was faint enhancement of cervical roots (Fig. 18.1) and also enhancing area at the lung apex (Fig. 18.2). Spinal cord was homogenous with no myelitis reported. This finding would agree with clinical symptoms and examination and also with assumed clinical diagnosis of HSV-2 polyradiculitis. Root enhancement on its own would not be as helpful as it can be seen in meningeal infiltration, CMV infection, Guillain-Barré, neurofibromas etc. but again clinical context provided the answer. Therefore this case was concluded as HSV 2 meningitis with associated radiculitis. Nerve conduction studies were not tolerated by patient. Patient’s symptoms improved on symptomatic treatment gradually and completely resolved over 6 months. Keywords  Radiculitis · Herpes simplex virus · Neuropathic pain

Hallmarks Lumbar puncture played essential and vital role in this case. HSV-2 radiculomyelitis is recognised clinical unit alongside herpetic meningitis and meningoencephalitis. Sacral presentation (and not dominantly cervical like this case) is known

Notes

123

under the name of Elsberg syndrome. In this particular case infection was suspected and treatment was initiated early. This is probably why this particular patient recovered without any clinical residuum/deficit. Please compare this case with case 8 (HSV-1 infection).

Further Reading Eberhardt O, Küker W, Dichgans J, Weller M. HSV-2 sacral radiculitis (Elsberg syndrome). Neurology. 2004;63:758–9. Ganzenmueller T, Karaguelle D, Schmitt C, Puppe W, Stachan-­ Kunstyr R, Bronzlik P, Sauerbrei A, Wegner F, Heim A. Prolonged detection of herpes simplex virus type 2 (HSV-2) DNA in cerebrospinal fluid despite antiviral therapy in a patient with HSV-2-­ associated radiculitis. Antivir Ther. 2012;17:125–8. Nadgir R, Yousem DM. Neuroradiology: the requisites. Philadelphia: Elsevier Health Sciences; 2016.

Notes

Chapter 19 Recurrent Otitis Media

63-year-old man was referred to the Neurology outpatient clinic for left sided headaches. He also complained about hearing problems on the left described as “muffled hearing”. The Clinical story was very interesting. Symptoms started 3  months ago with a pain around his left ear which then spread to the jaw. There was some facial numbness around his eye on the left side. There was also a pain behind the left ear. Effusion behind tympanic membrane was found so his headaches were originally assumed to be linked to a middle ear infection and an initial diagnosis of chronic otitis media was made. He underwent myringotomy and insertion of grommet. Patient felt instant relief and significantly better for a week but then his headache and the pain around the left ear reappeared. No cultures grew from effusion sent to microbiology. Patient was on three different antibiotics for 4 weeks with no effect. Ten days before neurology review he developed left facial nerve palsy, and a week later he developed diplopia especially when looking to the left. On objective examination left abducens, facial nerve palsy (lower motor neurone) and trigeminal/V nerve symptoms were found  – hypoasthesia (light touch) and allodynia (pin-prick), mainly V1 and V2 were involved (ophthalmic and maxillary, showed as white area). There was asymmetry of reflexes in legs, with increased reflexes on the right (ankle). There was no weakness of limbs and sphincter functions were normal. Cerebellar functions were normal. Pupils were symmetrical and reactive to light. © Springer Nature Switzerland AG 2019 O. Dolezal, Clinical Cases in Neurology, In Clinical Practice, https://doi.org/10.1007/978-3-030-16628-1_19

125

126

Chapter 19.  Recurrent Otitis Media Left abducens palsy Left LMN facial nerve palsy

+

+

5/5

5/5

+

+

+ + Neg. 5/5

+ Neg. 5/5

Thoughts Anatomically all symptoms seem to be limited to the left side therefore pathological process would likely be close to brainstem on the left (ipsilateral V, VI, VII nerve). A brainstem process would also explain contralateral right leg hyperre-

Thoughts

127

flexia (involvement of corticospinal tract before pyramid deccusation/crossing). As a chronic middle ear infection was present, infectious aetiology also comes to mind, but cultures were negative and there was no response to antibiotic treatment. The V and VI (plus III and IV) nerves are anatomically close as they pass through the cavernous sinus, therefore a focal lesion involving the left cavernous sinus (sinus is close to the pyramid and brainstem (VII nerve)) could also be a possibility. Recurrent effusion can be caused by Eustachian tube obstruction. In similar cases scanning is essential and an urgent MRI with gadolinium contrast was requested to see if there were any signs of meningeal enhancement. You would be also interested if there were any signs of any pathological processes in IAMs (cholesteatoma etc.), cavernous sinus (we included venogram) or pyramid.

Figure 19.1  MRI sagittal T1 weighted image with gadolinium contrast

128

Chapter 19.  Recurrent Otitis Media

Figure 19.2  MRI axial T1 weighted image with gadolinium contrast

Figure 19.3  MRI axial FLAIR image

 

Hallmarks

129

Key Symptoms Trigeminal nerve symptoms (V), abducens (VI) and facial nerve (VII) palsy

Answers First two scans (Figs. 19.1 and 19.2) are T1 weighted images with gadolinium showing enhancement of soft tissue/mass lesion in the left side of the nasopharynx measuring 24 × 17.5 mm in diameter (Fig. 19.1) with extension into the cavernous sinus and left temporal lobe with enhancement (Fig. 19.2). Third picture shows the lesion on FLAIR images (iso-intense to brain parenchyma, (Fig.  19.3)). The lesion was causing compression of the left trigeminal nerve anteriorly. Biopsy showed squamous cell carcinoma of nasopharyngeal origin (spreading intracranially and compressing Eustachian tube on the left and infiltrating left cavernous sinus). Radical surgery was not possible and radio and chemotherapy was arranged. Sadly the patient developed cavernous sinus thrombosis and died within 6  months of diagnosis. Keywords  Cavernous sinus · Facial nerve · Abducens nerve · Trigeminal nerve · Squamous cell carcinoma

Hallmarks This case again demonstrates intimate relationship between symptoms and anatomical location of lesion. Good anatomical knowledge narrowed down suspected pathologies and helped navigate radiologist better (see table below summarizing structures in cavernous sinus).

130

Chapter 19.  Recurrent Otitis Media

Structures in cavernous sinus III/oculomotor nerve

Symptoms (ipsilateral) Diplopia, mydriasis, ptosis

IV/trochlear nerve

Diplopia (when looking downwards)

V/trigeminal nerve (first and second division)

Neuralgia, sensory deficit

VI/abducens nerve

Diplopia when looking to affected side

Carotid artery

Occlusion/dissection can cause ischemic stroke

Internal carotid plexus (sympathetic pathways)

Miosis, partial ptosis (Horner’s syndrome)

Further Reading DeMyer W. Technique of the neurologic examination. 5th ed. New York: McGraw Hill Professional; 2003. Keane JR.  Cavernous sinus syndrome. Analysis of 151 cases. Arch Neurol. 1996;53:967–71. Preechawat P, Poonyathalang A, Boontantrapiwat S, Dhanachai M, Luxameechanporn T. Horner syndrome and abducens nerve paresis due to a paranasal sinus squamous cell carcinoma involving cavernous sinus. Neuroophthalmology. 2009;33:185–7. Ropper A, Samuels M, Klein J.  Adams and Victor’s principles of neurology. 10th ed. New York: McGraw Hill Professional; 2014. Zhu J-J, Padillo O, Duff J, Hsi B-L, Fletcher JA, Querfurth H. Cavernous sinus and leptomeningeal metastases arising from a squamous cell carcinoma of the face: case report. Neurosurgery. 2004;54:492–8; discussion 498–9.

Notes

Notes

131

Chapter 20 Deafness and Shoulder Weakness

67-year-old gentleman was referred to the neurology outpatient clinic by General Practitioner for weakness in his arms, especially shoulders. He also had difficulties holding his arms elevated or raising them above horizontal line. In his past medical history there was since youth bilateral longstanding hypoacusis. However it became more prominent on the right, alongside tinnitus, over last 2  years (progressing). Rest of medical history was unremarkable. On objective examination reflexes in arms were absent including triceps, biceps and brachioradialis. There was bilateral rather proximal (girdle) weakness (3/5) with some weakness distally (4/5) in upper extremities. Plantar responses were positive. Reflexes in legs were present (not increased). There were fasciculations in deltoids and obvious atrophy of trapezius and biceps bilaterally. Patient reported occasional bladder issues (retention/ delayed start). His hearing was very poor and the patient constantly shouted during encounter. Weber and Rinne tests responses were both reduced significantly, almost absent. Cerebellar functions were normal. Rest of cranial nerves and sensory functions were normal. Patient also complained about bilateral “buzzing” tinnitus.

© Springer Nature Switzerland AG 2019 O. Dolezal, Clinical Cases in Neurology, In Clinical Practice, https://doi.org/10.1007/978-3-030-16628-1_20

133

134

Chapter 20.  Deafness and Shoulder Weakness Bilateral hypoacusis

3/5

3/5





4/5

4/5

+

+

+ Pos. 5/5

+ Pos. 5/5

Thoughts In this case we have gentleman with longstanding but recently worsening symptom (hypoacusis). Weber and Rinne tests were almost without any response, which would be suggestive of sensorineural hearing loss. This is not particularly worrying as we know about patient’s hearing being problematic since childhood, but proximal weakness can be suggestive of myopathy or myositis. On the other hand con-

Thoughts

135

vincing fasciculations as a sign of denervation (alongside weakness) and a mixture of upper and lower motor neurone findings (positive plantars vs. muscle atrophy and areflexia) can be suggestive of anterior horncell degeneration/motor neurone disease (MND)/amyotrophic lateral sclerosis (ALS). Bladder issues/retention (if really neurogenic and not associated with e.g. prostate hyperplasia) would be rather typical for spinal cord lesion (myelopathy). Other options such as myasthenia gravis seemed unlikely (no ocular and bulbar involvement). Under circumstances blood tests were requested (Creatine kinase, Acetylcholine receptor and anti muscle-specific kinase antibodies). Nerve conduction studies/electromyography (NCS/EMG) seemed to be essential. MRI of cervical and thoracic spine was also requested (cervical radiculopathy, myelopathy). Hypoacusis and tinnitus are very non specific and longstanding findings but should not be ignored, especially as there is recent progression; MRI brain including IAMs was therefore arranged as well.

Figure 20.1  MRI coronal T2 weighted image

136

Chapter 20.  Deafness and Shoulder Weakness

Figure 20.2  MRI axial T2 weighted image

Figure 20.3  MRI axial T2 weighted image

 

Hallmarks

137

Key Symptoms Sensorineural hypoacusis, urinary retention, upper girdle muscles weakness/atrophy

Answers Electrophysiology (NCS/EMG) reported radicular motor involvement of C5, C6 and C7 roots bilaterally. This involvement would explain upper girdle findings. At that point there was a possibility of “man in a barrel” form of anterior horn cell degeneration (so dominantly affecting anterior horns in the cervical intumescence). Creatine kinase and myasthenia gravis serology were normal. MRI was very surprising as it found extensive superficial hemosiderosis (dark outline of the brain stem) (Figs.  20.1, 20.2, and 20.3), which could explain not only radiculopathic presentation (similar findings can be expected around root in cervical spine) but also would explain recent progression of hearing problems and tinnitus (Figs. 20.2 and 20.3). Superficial hemosiderosis of brain and cervical spine is very rare. It is usually caused by chronic blood leakage to the CSF.  Symptoms are usually progressive (sensorineural hearing loss, vestibular symptoms, cerebellar symptoms (ataxia), less commonly represented by radiculopathy as in this case). Whole spine and brain angiography was arranged including digital subtraction angiography (DSA) with no pathological finding (regarding dural fistula, AVM, cavernoma etc). Keywords Sensorineural hypoacusis · Radiculopathy · Superficial hemosiderosis

Hallmarks In this case it was very important to put each symptom into clinical context (progression hearing problems, proximal weakness, muscle atrophy, fasciculations etc.). It is not

138

Chapter 20.  Deafness and Shoulder Weakness

unusual in clinical practice to be hesitant even between relatively distant diagnoses, as during diagnostic process many options remained open (considering MND, myelopathy, radiculopathy, myopathy). Hearing issues could be overlooked as deemed longstanding but recent progression was suspicious. Proximal weakness would lead us to rather muscle process (myositis) and fasciculations to MND. Radiculopathy and other motor neuropathy can also lead to weakness, atrophy and fasciculations like in this case. Sphincter symptoms could be caused by lumbar and spinal radiculopathy but root hemosiderosis was not imaged in that region, so it could be deemed red herring. Plantars also played relatively misleading role (non specific) as there were no signs of myelopathy (upper motor neurone/corticospinal tract involvement).

Further Reading González-Usigli HA, Perez-Torres T, Saenz-Farret M, Rivero-­ Moragrega P.  Superficial siderosis misdiagnosed as idiopathic bilateral neurosensorial deafness. BMJ Case Rep. 2018; https:// doi.org/10.1136/bcr-2017-222639. Vale TC.  Idiopathic superficial siderosis. Arch Neurol. 2011;68: 1334. Wang J, Gong X.  Superficial siderosis of the CNS associated with multiple cerebral cavernous malformation. Neurology. 2009;72: 1187. Wilson D, Chatterjee F, Farmer SF, Rudge P, McCarron MO, Cowley P, Werring DJ.  Infratentorial superficial siderosis: c­ lassification, diagnostic criteria, and rational investigation pathway. Ann Neurol. 2017;81:333–43. Yoo A, Jou J, Klopfenstein JD, Kattah JC. Focused neuro-­otological review of superficial siderosis of the central nervous system. Front Neurol. 2018;9:358.

Notes

Notes

139

Chapter 21 Young Girl with the Wrist Drop

22-year-old young lady came to Neurology outpatient clinic with her mum and both were very distressed. She was referred by her General Practitioner with a 5 week history of sudden onset right hand weakness (she woke up with symptoms). Her grip strength was reduced and she had difficulties moving her wrist. She had pins and needles in the right forearm, right thumb and dorsum of her hand. She did not report any other symptoms. Initial clinical discussion was dominated by overwhelming fear that she was suffering from multiple sclerosis. Level of panic was aggravated by the fact that close relatives (mother’s sister) was diagnosed with multiple sclerosis some years ago. On objective examination obvious wrist drop was found and the brachioradialis muscle was weakened on the right side - there was reduced contraction and muscle bulk of this muscle during elbow flexion examination. Hand dorsal flexion (extension) was markedly weaker (arrow and 2/5 on diagram) on the right. Flexion of fingers was slightly weak but present. Wrist flexion, triceps and biceps strength were normal. There were no intrinsic muscles wasted. Sensory functions in her right hand had improved over the last 5 weeks; however this was not valid for motor recovery (only some improvement). Cranial nerves and cerebellar functions were completely normal and sphincter functions were intact. Reflexes were present and not particularly increased and plantars were negative. Shape of her feet was normal. Family history was as above but after some more detailed questioning patient © Springer Nature Switzerland AG 2019 O. Dolezal, Clinical Cases in Neurology, In Clinical Practice, https://doi.org/10.1007/978-3-030-16628-1_21

141

142

Chapter 21.  Young Girl with the Wrist Drop

mentioned that her father repeatedly had similar symptoms when he was younger repeatedly (wrist and foot drop), however he never asked for medical advice and is performing well without any disability (currently being in his mid 50s). CN, cerebellar normal

5/5

5/5

+

+

2/5 5/5

5/5

+

+

+ Neg. 5/5

+ Neg. 5/5

Thoughts Multiple sclerosis as a diagnosis dominated the discussion and it definitely put a lot of pressure on the doctor but was it a possible diagnosis here? Using anatomical knowledge it seemed that symptoms were present in radial nerve territory only, involving motor (wrist extensors – extensor carpi radialis brevis and longus) and sensory fibres as well. In demyelination you would

Thoughts

143

expect weakness to be rather global (throughout myotomes/ nerves as it involves white matter tracts in the brain or spine) and the same would be valid for sensory deficit (would be across dermatomes). Generally speaking the above findings were not suggestive of upper motor neurone aetiology. Reflexes were symmetrical, not increased; there was absence of other symptoms, and motor deficit in clearly defined muscles including brachioradialis muscle. So central nervous system process would be unlikely. Family history here is very interesting; however not because of demyelination but because of her father’s problems. Demyelination is not a genetic disease from “Mendelian” point of view but we should consider possibility of hereditary neuropathy. The most “famous” hereditary neuropathy Charcot-Marietooth (hereditary motor sensory neuropathy HMSN) is unlikely (no convincing disability in the family, no pes cavus, sensory symptoms present etc.). Most likely diagnosis here would be hereditary neuropathy with tendency to pressure palsies (HNPP)

Figure 21.1  MRI axial FLAIR image

144

Chapter 21.  Young Girl with the Wrist Drop

as she is slowly recovering. MRI brain and cervical spine (see below) to relieve rather social and emotional pressures and more importantly nerve conduction studies (NCS) and bloods were requested. Bloods were sent to genetics and for polyneuropathy screen (folate, B12, vasculitis screen, glycaemia, HbA1 etc.). Key Symptoms Radial nerve palsy (wrist drop) with corresponding sensory deficit

Answers MRI scan of the brain and cervical spine was unsurprisingly within normal limits. There was tiny frontal hyperintensity on the right which is an irrelevant finding (Fig.  21.1). Vasculitis screen, Lyme disease and the polyneuropathy screen were normal. Genetic examination was positive for PMP22 mutation  – heterozygous positive (deletion on 17p12). Eventually a sample was obtained from father with the same result. Nerve conduction studies supported the diagnosis. The diagnosis of hereditary neuropathy with tendency to pressure palsies was confirmed. Keywords  Radial nerve palsy · Hereditary polyneuropathy · Pressure palsies · PMP22

Hallmarks In similar cases clinicians can face very difficult situations. Often when patients mind is firmly set on one particular diagnosis it is difficult to shift their opinion. In this case it was important not to “fall into the trap” of demyelination. In this scenario clinician somehow failed to resist social pressure, however it is questionable if requesting MRI was completely incorrect. It is true that MS is the most likely neurological disease in younger adults, but clinicians should insist on validity of objective examination, which alongside the family history proved to be the most helpful and decisive factor in this case.

Notes

145

Further Reading Bird TD. Hereditary neuropathy with liability to pressure palsies. In: GeneReviews. Seattle: University of Washington; 1998. DeMyer W. Technique of the neurologic examination. 5th ed. New York: McGraw Hill Professional; 2003. Dyck PJ, Thomas PK.  Peripheral neuropathy (2-vol. set). Schweiz Arch Neurol Psychiatr. 2006;157:81.

Notes

Chapter 22 Facial Tingling and Poor Hearing

55-year-old lady came to clinic complaining about several months history of paresthesias (tingling) affecting left side of her face and tongue. Symptoms came on gradually. She also noticed a reduction in her sense of taste. There was no facial asymmetry. She suffered from migraines in the past, which have become more frequent lately (occurring on a weekly basis). During a migraine the pain was generally situated behind the right eye (contralateral side to paresthesias/tingling). Patient was very philosophical about her symptoms, explaining them as complicated migraine and “definitely linked to her migraines”. On objective examination reduced pinprick and light touch sensation within the first, second and third left trigeminal nerve division (white area) was found. There was hypoacusis (Rinne and Weber tests were both reduced) on the left. Rest of cranial nerves looked normal. Cerebellar functions were normal. She possibly felt less inner cheek (mucosa) on the left and left side of her tongue when touched by spatula. Muscles of mastication and facial muscles were normal. Taste was not tested in the clinic (relying only on subjective information it seemed reduced). Reflexes were brisk, slightly increased in legs but otherwise there was no other focal symptomatology.

© Springer Nature Switzerland AG 2019 O. Dolezal, Clinical Cases in Neurology, In Clinical Practice, https://doi.org/10.1007/978-3-030-16628-1_22

147

148

Chapter 22.  Facial Tingling and Poor Hearing CN and cerebellar normal, except left hemifacial hypoasthesia and sensoryneural hypoacusis

+

+

5/5

5/5

+ +

+ +

+ + Neg. 5/5

+ + Neg. 5/5

Thoughts In this case sensory symptoms are chronic but completely new to patient. They can represent complicated migraine but they do not seem to be directly connected to her intermittent headaches. Anatomical knowledge is essential here. Taste is covered by facial nerve (chorda tympani) and

Thoughts

149

sensation from the tongue is carried by trigeminal nerve (lingual nerve branch of mandibular (V3) nerve). It seemed that other trigeminal divisions (V1 and V2) are involved as well (tingling). Newly found hearing issues on the left were likely sensorineural (Rinne and Weber tests) so left VIII/vestibulochochlear nerve (or brainstem) involvement was expected. Pontocerebellar angle process should also be considered (especially VII and VIII nerve are in very intimate relationship). MRI was requested to rule out AVM, aneurysm, vestibular Schwannoma/neurinoma or meningioma.

Figure 22.1  MRI axial FLAIR image

150

Chapter 22.  Facial Tingling and Poor Hearing

Figure 22.2  MRI axial FLAIR image

Figure 22.3  MRI coronal T1 weighted image without contrast

 

Answers

151

Figure 22.4  MRI coronal T1 weighted image with gadolinium contrast

Key Symptoms Hypoacusis (sensorineural), trigeminal symptoms, reduced taste (hypogeusia)

Answers MRI found a soft tissue mass measuring 1.4 × 1.4 × 2 cm in the left pontocerebellar angle, anteriorly in contact with left trigeminal nerve. Laterally it was expanding into IAM in depth about 6.5 mm. First two scans are not very helpful as lesion is isointense on FLAIR images (Figs.  22.1 and 22.2) (better seen on T1 non contrast (Fig. 22.3) scan) but you can clearly see asymmetry in brainstem structure. There was also inhomogeneous contrast enhancement on T1 Gadolinium scan (Fig.  22.4). Findings are suggestive of vestibular

152

Chapter 22.  Facial Tingling and Poor Hearing

Schwannoma. Patient was referred to neurosurgery. After surgery there was some improvement especially of sensory facial symptoms but her hearing got worse on the left (likely due to inevitable damage to functioning remnants of VIIIth nerve during removal). Hearing is postoperatively preserved in only around one third of patients. Keywords  Sensorineural hypoacusis · Trigeminal nerve · Facial hypoasthesia · Vestibular Schwannoma

Hallmarks It can seem that MRI findings again answered all questions but nothing is more further from the truth. Neurological examination and anatomical knowledge led us to ­appropriate choice of test. Patient’s symptoms were very mild in intensity and examination identified some of the symptoms that even the patient was not aware of (hypoacusis). Early diagnosis in similar (relatively benign) lesions is essential as the size of tumour and extent of infiltration of neighbouring structures is crucial regarding surgery outcome.

Further Reading Agur AMR, Dalley AF, Grant JCB.  Grant’s atlas of anatomy. Philadelphia: Lippincott Williams & Wilkins; 2013. Nadgir R, Yousem DM. Neuroradiology: the requisites. Philadelphia: Elsevier Health Sciences; 2016. Ropper A, Samuels M, Klein J.  Adams and Victor’s principles of neurology. 10th ed. New York: McGraw Hill Professional; 2014.

 

Notes

Notes

153

Chapter 23 Balance and Gait Problems

73-year-old gentleman was referred to Neurology outpatient clinic by his General Practitioner for gait and balance issues. Patient reported a 3 year history of difficulty mobilising due to leg weakness, as well as his problems with balance. He also mentioned memory problems, which was corroborated by his wife. His symptoms had progressively worsened over the years. He also complained of proximal limb weakness, worse on the left, long standing urinary problems (urgency with occasional “incidents”), intermittent faecal incontinence and lower back pain, not-radiating to his legs. He also reported significant weight loss despite a good appetite. His swallowing had become of some concern recently as well due to occasionally choking. He struggled mainly with liquids. His symptoms seemed to worsen as the day progressed (aggravated by fatigue/exertion). On examination he had a broad based gait, decreased power in legs, with a grade of 4/5 on the right and 3/5 on the left. There was some loss of dexterity in his hands when picking up small things (coins, pencil etc.) but muscle strength seemed normal. There were increased reflexes in lower extremities and normal reflexes in upper extremities. He had bilateral nystagmus with diplopia in lateral position. There was also finger/nose and finger/finger ataxia with dysmetria and intention tremor. He struggled to drink (choked on examination) but soft palate and tongue moved well. His

© Springer Nature Switzerland AG 2019 O. Dolezal, Clinical Cases in Neurology, In Clinical Practice, https://doi.org/10.1007/978-3-030-16628-1_23

155

156

Chapter 23.  Balance and Gait Problems

speech seemed slurred/dysarthric, but according to his family it had not progressed recently. His jaw jerk was increased. There was no facial asymmetry (no ptosis or facial drooping). Bilateral nystagmus, ataxia Bulbar symptoms

5/5

5/5

+

+

5/5

5/5

++

++

++ Pos. 4/5

++ Pos. 3/5

Thoughts To summarise this patient’s complex symptoms; there was spastic paraparesis, slow cognitive pace, incontinence (could be overflow so linked to retention), ataxia and pseudobulbar symptoms (swallowing, speech difficulties). We are dealing here with a progressive disorder involving not only motor

Thoughts

157

functions and sphincters but cognitive functions as well. As there is ataxia there is likely a lesion in posterior fossa with involvement of not only cerebellum but corticospinal tracts and brainstem in general. Cognitive issues are relatively surprising but hydrocephalus could be responsible for patient decline (triad of gait disturbance, cognitive issues and incontinence, for more see case 14). Another option here would be a degenerative disorder (e.g. amyotrophic lateral sclerosis with frontotemporal dementia (ALS/FTD complex). Patient’s fatigability raises the question of myasthenia gravis but lack of other symptoms, especially ocular, upper motor neurone nature of his paraparesis, ataxia and sphincter issues would not be suggestive of this. To rule out an organic lesion an MRI of the brain was arranged.

Figure 23.1  MRI sagittal T1 weighted image with gadolinium contrast

158

Chapter 23.  Balance and Gait Problems

Figure 23.2  MRI axial T1 weighted image with gadolinium contrast

Figure 23.3  MRI coronal FLAIR image

 Answers

159

Figure 23.4  MRI sagittal FLAIR image

Key Symptoms Ataxia, upper motor neurone paraparesis, incontinence, dysphagia, dysarthria, cognitive deficit/memory impairment

Answers MRI of the brain found a large tumour mass filling the foramen magnum, extending into jugular foramen and condylar canal with a small erosion of posterior aspect of the right occipital condyle (Figs. 23.1, 23.2, and 23.4). There were initial signs of hydrocephalus (three ventricles dilated) (Fig.  23.3). Lesion was enhancing after gadolinium contrast (Figs. 23.1 and 23.2). Schwannoma or meningioma were in differential diagnosis (plasmocytoma was also considered). Before we were able to conclude the diagnosis, to consider or perform biopsy, the patient unfortunately suffered a severe myocardial infarction and after angioplasty he was started on anticoagulation. The patient eventually declined proposed biopsy and subsequent

160

Chapter 23.  Balance and Gait Problems

radiotherapy and followed a palliative route. He unfortunately passed away within 6 months of his first neurology review. Keywords  Paraparesis · Ataxia · Bulbar symptoms · Cognitive decline · Incontinence · Hydrocephalus · Posterior fossa · Meningioma

Hallmarks Objective examination again helped to identify the most likely location of the lesion. This case also demonstrated that we are not always able to conclude cases definitively. Informing the patient fully about possibilities, risks and likely prognosis (especially when done in multidisciplinary fashion and in this case after discussions with neuroncologist, neurologist, neurosurgeon) provides patient with the power to change route of diagnostic and therapeutic measures. Respecting patient’s wish is then much easier for treating doctor and the patient’s family as well.

Further Reading Kaye AH.  Posterior fossa meningiomas in Sindou, Marc. Practical handbook of neurosurgery: from leading neurosurgeons. Wein New York: Springer-Verlag, 2009. Print. Nadgir R, Yousem DM.  Neuroradiology: the requisites. Mosby, Philadelphia: Elsevier Health Sciences; 2016.

Notes

Chapter 24 Numb Legs and Constipation

70-year-old man came to Neurology outpatient clinic with symptoms he had had for approximately 12 months. Symptoms were fluctuating with “good days and bad days”. He reported “weird feeling” in both his legs. He described them as being numb as he “could not feel them properly and they were occasionally weaker”. He also had constipation and significantly delayed start of urination. He did not have any headaches. There was also pain in his right shoulder. There was previous history of sciatica 20 years ago (but according to patient current problems did not seem similar in any way). On objective examination reflexes were quite brisk in upper extremities. In lower extremities patellar reflex was increased on the left, normal on the right. Ankle reflexes were absent bilaterally. His right leg was weaker (4+/5) with normal strength (5/5) on the left. There was thermoception deficit on the right with normal pain and light touch perception. Proprioception was reduced on the right. On the left there was reduced pain, light touch and thermoception but proprioception seemed normal. Sensory functions in upper extremities and torso looked normal. Plantars were neutral/negative. There was urinary retention. Cranial nerves and cerebellar functions were normal. There were not any cognitive or memory issues.

© Springer Nature Switzerland AG 2019 O. Dolezal, Clinical Cases in Neurology, In Clinical Practice, https://doi.org/10.1007/978-3-030-16628-1_24

161

162

Chapter 24.  Numb Legs and Constipation CN, cerebellar functions normal

5/5

5/5

+ +

+ +

5/5

5/5

+ +

+

– Neg 4+/5

– Neg 5/5

Thoughts It seemed that there was incomplete syringomyelic sensory dissociation. Usually there is dissociation between light touch, proprioception and pain and thermoception/temperature (not the case here). Symptoms were likely more prevalent on the right side (weakness). There were only questionable symptoms in upper extremities (hyperreflexia) and lack of other supratentorial and brainstem symptoms very much narrow our possibilities. Therefore his complex symptoms/signs can represent a lesion in the cord, most likely in lower thoracic or lumbar segments (below cervical intumescence).

Thoughts

163

Myelitis or myelopathy should also be considered. Syringomyelia or “hemi-cord” lesion (likely on the right because of ipsilateral weakness) could also be a culprit here especially because of various sensory deficits in different modalities but as mentioned, clinical picture of dissociation is not fully expressed. Hypo/areflexia distally in legs and negative plantars is surprising and raises the possibility of participation of polyneuropathy. Also, fluctuation of symptoms is interesting and would be atypical for cord compression or cord stroke. Demyelination can be typically relapsing remitting but age and also intensity of oscillations (“good days and bad days”) would be unlikely. There also could be Chiari malformation (leading to syringomyelia) or posterior fossa tumours participating. MRI of the cervical and thoracic spine seemed to be a logical choice here as well as MRI brain. Lumbar puncture was also performed.

Figure 24.1 MRI sagittal T1 weighted image without gadolinium contrast

164

Chapter 24.  Numb Legs and Constipation

Figure 24.2  MRI sagittal T2 weighted image

Key Symptoms Fluctuating paraparesis, sphincter symptoms/retention, sensory deficit in legs (incomplete dissociation)

Answers CSF analysis found high protein of 1.2 g/l with normal cells. MRI of brain was normal with no signs of process in posterior fossa or cervical spine. Pituitary tumour (likely adenoma) which was clearly coincidental was noted (Fig.  24.1). But MRI of whole spine found lower cord oedema (seen as white hyperintense region on Fig. 24.2). Above that lesion you can see dark flow voids on the anterior surface of the cord (Fig. 24.2). This was suggestive of a spinal AVM or dural arteriovenous (A-V) fistula. Digital subtraction angiography (DSA) is method of choice in similar cases. Dural arteriovenous fistula (DAVF) was established by DSA and was

Notes

165

embolised. Post operative angiogram confirmed the occlusion at of the left L3 radicular vessel; right L3 vessel was not possible to catheterise. Patient clinically improved after intervention and remained stable (mobilising with two ­ crutches and using wheelchair for longer distances). Keywords  Sphincter symptoms · Paraparesis · Dural arterio­ venous fistula

Hallmarks MRI helped us here massively to localise the lesion and also identifying primary cause but neurological examination narrowed down our possibilities. Fluctuating symptoms suddenly made more sense as it would be typical for dural fistula.

Further Reading Lanzino G. Cavernous malformations of the brain and spinal cord. New York: Thieme; 2011. Lindsay KW, Bone I, Fuller G.  Neurology and neurosurgery illustrated e-book. St. Louis: Elsevier Health Sciences; 2010. Schick U, Hassler W. Treatment and outcome of spinal dural arteriovenous fistulas. Eur Spine J. 2003;12:350–5.

Notes

Chapter 25 Yellow and Weak Young Man

19-year-old man was admitted to hospital with left arm weakness and altered sensation as the main complaint lasting for 24 hours and a 5 day history of altered sensation in both feet and weakness in his legs. The day before admission he realised that he could not lift his left arm above his head. He also had pins and needles in his fingertips bilaterally. He denied any head injury or other trauma. He had no recent illness including diarrhoea and vomiting. He had felt fatigued and exhausted for last 4  weeks. On examination there was bilateral ankle weakness (4+/5) with foot drop, more prominent on the right (4/5). He was not able to lift his left arm over his head (2/5 proximally). He also had some difficulties lifting his right arm over his head (especially against resistance  – 3/5). There was some ataxia and dysmetria (during finger/nose testing) but likely due to weakness. Reflexes were absent in all four extremities. Cranial nerves were normal. He was not breathless during physical activity. Bulbar functions were normal. There was rather distal light touch hypoasthesia but rest of the sensory functions were normal (vibration sense, pain). His complexion was slightly pale and yellowish. He had a temperature of 37.6 °C.

© Springer Nature Switzerland AG 2019 O. Dolezal, Clinical Cases in Neurology, In Clinical Practice, https://doi.org/10.1007/978-3-030-16628-1_25

167

168

Chapter 25.  Yellow and Weak Young Man CN, cerebellar functions normal

3/5

2/5





5/5

4+/5



4/5



– Neg. 4/5

4+/5

– Neg. 4+/5

Thoughts In this case we are dealing with an otherwise healthy gentleman with no significant medical history. His yellowish colour can be habitual but can also represent jaundice/icterus. Temperature was increased, so infectious aetiology should be considered. From a neurological point of view his symptoms seem to be progressive, initially involving his legs and eventually spreading to his arms. So there is ascending weakness with peripheral sensory symptoms. His weakness is obviously lower motor neurone in nature. Autoimmune inflammatory demyelinating neuropathy (polyradiculoneuropathy)/GuillainBarré syndrome comes to mind, but a spinal cord lesion at the stage of spinal shock can have lower motor neurone signs as well. So despite there is no collateral history of trauma, a cervical spine lesion need to be ruled out. Demyelination is

 

Hallmarks

169

unlikely. Blood tests (namely LFTs), lumbar puncture, ultrasound of abdomen and MRI of cervical spine were arranged.

Answers MRI of cervical spine was unremarkable (not shown). ALT serum level was 254  iu/l (range 1–50  iu/l), billirubin was 41 umol/l (0–21 umol/l), CRP was